Tuesday, March 8, 2011

Court says Dragon speed limit is 65 mph


photo by the Dragonater

UPDATE: $65-Million class action lawsuit filed against police by attorney for The Dragonater

Sticky: Trucker given immunity for killing biker, no ticket no arrest for crossing centerline and blocking both lanes on the Dragon

UPDATE: Governor Haslam orders THP to rape all motorists in Tennessee

TN cops arrested for arresting biker for no driver license and no vehicle registration (none required by law)

Thank's to the Tennessee Attorney General biker who reads this blog...here's your sign (that 99% of your job is based on a lie). Would you rather pay a $250 speeding ticket on the Dragon, or download the govt documents on this webpage and pay a lawyer $100 to get the ticket dismissed without trial, or DIY for free?

TN Code 55-8-152 - Speed limits - penalties.
(a) Except as provided in subsection (c), it is unlawful for any person to operate or drive a motor vehicle upon any highway or public road of this state in excess of sixty-five miles per hour (65 mph).

TN Code 55-8-153 Establishment of Speed Zones.
(a) The department of transportation is empowered to lower the speed limits prescribed in § 55-8-152 in business, urban or residential districts, or at any congested area, dangerous intersection or whenever and wherever the department shall determine, upon the basis of an engineering and traffic investigation, that the public safety requires a lower speed limit.

"The defendant asserted that the posted speed limit on State Route 62 had not been established in compliance with applicable law. The speeding citation in this case specifically charges that the defendant committed the offense of speeding at 67 mph in 45 mph zone. If the posted speed sign was placed without statutory authority, the failure of a motorist to heed its restrictions could be negligence but he should not be penalized with the consequences of the rule of negligence per se of violating a statute, since a statutorily established speed limit prevails over speed signs erected without statutory authority. The Virginia court stated as follows: A city traffic engineer's proffered testimony tended to prove, although not conclusively establish, that no traffic or engineering study had been performed as required by Code 46.2-1300 in order to establish a thirty-mile-per-hour speed limit. The judgment of the trial court is hereby vacated, and this case is remanded to the court below for further proceedings in accordance with this opinion. Costs on appeal are taxed to the City of Oak Ridge."
-COURT OF APPEALS OF TENNESSEE AT KNOXVILLE, CITY OF OAK RIDGE v. DIANA RUTH BROWN, No. E2008-02219-COA-R3-CV, MAY 8, 2009

QUESTION: "Is an engineering study required for posting speed limits?"
ANSWER: "It depends. Maximum statewide speed limits are established by state legislatures according to road class (e.g., Interstate highways) and geographic area (e.g., rural vs. urban areas). The legislated maximum speed limit generally applies to all roads of a particular class throughout the State. This is referred to as a statutory maximum speed limit, which applies "unless otherwise posted" and above which a speed limit cannot be legally posted. For example, the statutory maximum speed limit for rural freeways in a given State might be 65 MPH. No engineering study would be needed to post a 65 MPH speed limit on a rural Interstate highway in that State, and even if an engineering study indicated that 75 MPH might be a more appropriate speed for the conditions, the statutory maximum would prohibit the State from posting any limit higher than 65 MPH. Similarly, statutory maximum limits are often legislated for urban streets within city limits, such as 30 MPH. However, State and local governments typically have the authority to change the limits by establishing speed zones, with posted speed limits lower than the statutory maximum, for highway or street sections where statutory limits do not fit specific road or traffic conditions. An engineering study is required for setting the limit for altered speed zones. The engineering study takes into consideration such factors as operating speeds of free-flowing traffic, crash experience, roadside development, roadway geometry, parking, and pedestrian traffic."
-Federal Highway Administration, U.S. Dept of Transportation, Frequently Asked Questions - Part 2 - Signs, Manual on Uniform Traffic Control Devices 2011

Section 2B.13 Speed Limit Sign (R2-1)
Standard: After an engineering study has been made inaccordance with established traffic engineering practices, the Speed Limit (R2-1) sign (see Figure 2B-1) shall display the limit established by law, ordinance, regulation, or as adopted by the authorized agency. The speed limits shown shall be in multiples of 10 km/h or 5 mph.
Guidance: At least once every 5 years, States and local agencies should reevaluate non-statutory speed limits on segments of their roadways that have undergone a significant change in roadway characteristics or surrounding land use since the last review."
-Federal Highway Administration, U.S. Dept of Transportation, Chapter 2B. Regulatory Signs, Manual on Uniform Traffic Control Devices 2011

"The designer responsible for a signing and/or pavement marking project should be aware that the design must comply with various standards. In addition to Department Standard Specifications, the following standards should be consulted: Manual on Uniform Traffic Control Devices – The MUTCD is therefore the basic guide for signing and marking. The requirements of the MUTCD must be met, as a minimum, on all roads in Tennessee. All regulatory and warning signs shall meet the design and installation requirements of the MUTCD. The MUTCD is an ever changing standard. It is constantly being updated and revised. It is important to keep up-to-date with the latest requirements and compliance dates of the MUTCD."
-Tennessee Dept of Transporation (TDOT), CHAPTER 6 - SIGNING AND PAVEMENT MARKINGS, TDOT Traffic Design Manual (December 2003) (PDF, 4.47MB)

Tennessee MUTCD State Information as of Nov 2011
Status of the National MUTCD (2003 Edition)
State MUTCD Not Applicable
State Supplement Not Applicable
-Federal Highway Administration, U.S. Dept of Transportation, Manual on Uniform Traffic Control Devices 2011

State Traffic Control Detailed Drawings, Policies, Design Manuals, etc.
Roadway Standard Drawings
TDOT Traffic Design Manual (December 2003) (PDF, 4.47MB)

Adoption Status of 2009 National MUTCD by States
Tennessee - NO, January 2012 Adoption of the national MUTCD along with a State supplement(s)
-Federal Highway Administration, U.S. Dept of Transportation, Manual on Uniform Traffic Control Devices 2011

"Dear Mr. Lee: Please find enclosed the Road Safety Audit Review of U. S. 129 from North Carolina State Line to Tabcat Bridge in Blount County, dated July 25, 2007. We have no record of a Traffic Engineering Survey and/or Speed Audit for this location."
-Marion E. Hilt, Staff Attorney, TDOT, Tennessee Open Records Act Request, February 23, 2011

"The case is dismissed nolle prosequi, with prejudice. Without a Traffic Engineering Speed Audit, the default speed limit is 65 MPH."
-Mr. X, Blount County Public Defenders Office, attorney for The Dragonater, outside of Blount County General Sessions Court after the Preliminary Examination on probable cause, 7 February 2011

All references to "Mr. X" -- the name of the "public defender" lawyer -- have been deleted from this blog post, per written request of said public defender, under threat from the Tennessee Attorney General, dated 5 April 2011, received 14 May 2011. After all, it would be bad for business for people to hear that a public defender actually won a case (a fireable offense), that all cops in Blount County are nothing more than armed robbers perping dozens of felony carjackings every day, and for the sheeple to learn how to actually win in traffic court, especially on The Dragon. And there is no better to way to win than dismissal with prejudice before any hearing or trial, without a single word of testimony spoken in court. Defense lawyers, prosecutors and judges are "officers of the Court". The prosecutor dismissed this case nolle prosequi before trial, in sidebar "off the record" with no transcript allowed, without a court order of dismissal, and it is now expunged. So this case is not a published opinion by a judge that can be cited in future cases. Which is exactly why it was dismissed without trial. But that does not change the fact that this same defense is used to win 1,000s of trials in traffic courts all over USA, and YOU can use it too.

UPDATE: "And finally, you may notice some big purple devices mounted up in the trees about every couple of miles as you head toward and through the Dragon. They are temporary speed cameras that are being used in a study to determine the best locations for permanent units. Stay tuned as we will pass along any further information we receive..."
-Killboy.com, And A Couple More Things, 1 April 2011


by John Lee, a/k/a The Dragonater

DEALS GAP, TENN. -- The old Triumph Daytona 600 never felt right, even when new -- suspension too harsh no matter what the shock settings, easier to turn left than right, chain noise. I set the static sag, bump and rebound based on SportRider Mag's setup, adjusted the triple trees for the front ride height change from Pirelli Diablo triangular profile tyres to Dunlop's round profile, no rear ride hight adjuster so used rear preload to change ride height and modify turn in. When I checked the wheel alignment, the rear wheel was offset 1/2 inch from centerline, with no way to adjust, a common problem with many bikes.

So for safety's sake, I finally invested an afternoon getting a chassis setup at Wheelers Performance in Robbinsville NC.

Suspension sag check...no change.

Springs: OK to rider weight, no change.

Preload: OK, no change.

Bump/rebound: 1 click increase to front bump.

Rear wheel alignment...big change. Start: front and rear tires aligned with each other, but not aligned with chassis centerline. Finish: tires no longer aligned to each other, but now parallel to chassis centerline (factory setting).


Cornering the hairpins at the Gap is a problem for me, partly because I'm a wimp, and partly due to sudden throttle on acceleration from idle and suicidal throttle response below 6500 RPM (safety defect mandated by EPA's Green Nazis). Wheeler's price for dyno tune and engine remap is about $500, which requires purchase of a $200 ECU key from the factory for Tuneboy, or free key from TuneECU freeware. Ken recommended trying the factory dealer first, requiring payment of 1/2 hour shop time before even checking to see if a factory generic remap existed ($45, no remap existed, ECU FAIL closed loop status, no dyno for non-factory remap available). I already replaced and upgraded the entire charging system to Japanese, after the factory failed at 2,000 miles, so not much faith in the stealers.

So I began my ride home to Ktown on a "new" bike. Only a minor change in front bump setting, so maybe it would turn in a little better on the brakes? Nothing radical. But setting the rear alignment back to factory baseline meant the problem with right-turns would feel worse. So the point of the ride was to just get a feel for the "radical" changes without crashing, not set a "lap record" (impossible for me anyway), and to motivate myself to invest up to $500 in an engine remap, which would make the Dragon much more enjoyable, and all roads safer.

At Mile Marker 0.5 Northbound, I rounded a curve and was confronted by a THP state trooper standing in the middle of the road, flailing his arms wildly, pointing at me to stop. Odd, since my radar detector never barked. Guess I shoulda read the Speed Trap Map.

Video of the traffic stop, covert video by the Dragonater:



There went my $500 for Wheeler to improve the safety of my bike. Either I pay the $250 speeding ticket for alleged 60 in a 30 zone, or I invest $$$ in fighting the ticket.

In a previous conversation I had with the supervisor of the National Park Service at Sugarlands HQ in Gatlinburg, the official map of the Great Smokey Mountain National Park showed that Federal jurisdiction on the Dragon is measured on the centerline of US129. So I knew a THP trooper has zero state jurisdiction to run a speed trap at Mile Marker 0.5, and that a state court in Blount County had zero jurisdiction to handle a traffic ticket trial. I also knew there was a 99.9% probability there was no Traffic Engineering Survey performed by TDOT, as required by law to set a speed limit of 30 MPH on the Dragon. GAME ON.

Photo of Federal Park Ranger writing tickets on the Dragon? May 2011


The Battle Of Deals Gap: Photo of Southbound lane at Tabcat Bridge Mile Marker 11 on the Dragon

Once Upon A Time On The Dragon...there were no deaths with a 55 speed limit. Then the limit was illegally dropped to 40 and the killing began. The posted limit was changed again to 30 MPH in 2002, under pencil-whipped orders from Nashville, and the death rate increased again. When US129 was closed completely in 2010, the death rate doubled again, despite a 90% drop in traffic. That's why it's DANGEROUS and ILLEGAL to reduce a speed limit without a Traffic Engineering Speed Audit by TN Dept of Transportation, as REQUIRED by TN Code. This MANDATORY Speed Audit was NEVER done, as confessed in writing by TDOT.

This means that THERE IS NO VALID SPEED LIMIT POSTED ON THE DRAGON...SO THE REAL-WORLD SPEED LIMIT REVERTS TO THE DEFAULT SPEED LIMIT IN TN CODE...65 MPH. Don't believe me, read THE LAW for yourself, then THINK for yourself. THP even admits it has no jurisdiction on major portions of the Dragon, including THP's favorite speed traps, due to Foothills Land Conservancy giving 5,000 acres to the federal Park Service. Take ACTION if and when you comprehend the magnitude of the DEADLY FRAUD perpetrated upon you by Big Brother...which is FELONY OFFICIAL OPPRESSION under TN Code.

The Dragonater was appointed a "public defender" for defending this alleged speeding complaint. The Preliminary Exam hearing was held in Blount County General Sessions Court. By the start of docket on the day of the hearing, the public defender, "Mr. X", had not returned phones calls nor responded to registered letters. When The Dragonater attempted to actually SPEAK to his public defender for the first time, in the court room before the hearing, Judge Brewer SCREAMED that he would have The Dragonater ARRESTED for attempting such a crime. The judge's job is to convict all defendants, and that's impossible to do if defendants were allowed to actually speak to their lawyers. So The Dragonater was not allowed to speak to "his" lawyer, nor was he allowed to leave the courtroom to gather additional evidence from his vehicle.

"Sober as a judge."
-Charles Lamb


After a few minutes, the public defender left the courtroom, and did "order" the bailiffs to allow The Dragonater to leave with him. While conferring in a meeting room, The Dragonater advised his lawyer of the additional evidence obtained during his TN Open Records Act Requests from THP and TDOT -- Sgt Huckeby's personnel file, THP Stalker radar operator manual, TDOT Safety Audit Review Report banning all business and journalism from Deals Gap, TDOT letters admitting no Traffic Engineering Speed Audit was performed on US129 at Deals Gap, The Dragonater's undercover video of the traffic stop, and The Dragonater's undercover video of Sgt Huckeby speeding on the Dragon.

Video of Sgt Huckeby speeding on the Dragon, caught on tape by The Dragonater:



Within a few minutes of The Dragonater handing his lawyer this evidence, the Blount County assistant attorney general moved the court for Nolle Prosequi, dismissing all charges WITH PREJUDICE, with COURT COSTS PAID BY THE STATE.

Video interview of John Lee at Preliminary Examination hearing in Blount County general sessions court:



The Dragonater won again in Blount County traffic court! The Blount County attorney general (an "officer of THE COURT") refused to prosecute a 60 MPH speeding ticket on the Dragon. According to Blount County Public Defender "Mr. X" (an "officer of THE COURT"), based on the lack of mandatory Traffic Engineering Survey Speed Audit by TDOT, the speed limit on the Dragon at Deals Gap now officially defaults to 65 MPH, as required by TN Code. Even a TN state trooper admitted on video that the Highway Patrol, Blount County deputy sheriffs and Blount County courts have no jurisdiction on portions (or all) of the Dragon, that it's now Federal jurisdiction. Judge Brewer (an "officer of THE COURT") promptly ordered this case dismissed "off the record" -- so fast it's not even on the court reporter's transcript ("bench conference" without defendant present at the "bar").

Proving the speed limit is 65 on the Dragon, by John Lee host of the Pirate News Radio Show on Pirate News TV:

Listen to the Pirate News Radio Show at 5pm Tuesday on WBCR 1470am for the latest news on this courtroom victory by The Dragonater. Worldwide live radio stream. The radio show is rebroadcast worldwide on Pirate News TV, and in Knox County and parts of Blount County on Charter Channel 6, Knology Channel 6, Comcast Channel 12 and ATT Uverse Channel 99. De Facto Speed Limit on the Dragon
de facto. This phrase is used to characterize an officer, a government, a past action, or a state of affairs that must be accepted for all practical purposes, but is illegal or illegitimate. Thus, an office, position, or status existing under a claim or color of right, such as a de facto corporation. In this sense it is the contrary of de jure, which means rightful, legitimate, just, or constitutional. Thus, an officer, king, or government de facto is one that is in actual possession of the office or supreme power, but by usurpation, or without lawful title; while an officer, king, or governor de jure is one who has just claim and rightful title to the office or power, but has never had plenary possession of it, or is not in actual possession. A wife de facto is one whose marriage is Voidable by decree, as distinguished from a wife de jure, or lawful wife. But the term is also frequently used independently of any distinction from de jure; thus a blockade de facto is a blockade that is actually maintained, as distinguished from a mere paper blockade. -West's Encyclopedia of American Law, edition 2 DE FACTO. i. e. in deed. A term used to denote a thing actually done; a president of the United States de facto is one in the exercise of the executive power, and is distinguished from one, who being legally entitled to such power is ejected from it; the latter would be a president de jure. An officer de facto is frequently considered as an officer de jure, and his official acts are of equal validity. 10 S. & R. 250; 4 Binn. R. 371; 11 S. & R. 411, 414; Coxe, 318; 9 Mass. 231; 10 Mass. 290; 15 Mass. 180; 5 Pick. 487. -Bouvier Law Dictionary, 1856 Color of Law. The appearance of a legal right. The act of a state officer, regardless of whether or not the act is within the limits of his or her authority, is considered an act under color of law if the officer purports to be conducting himself or herself in the course of official duties. Under the civil rights act of 1871 (42 U.S.C.A. Section 1983), color of law is synonymous with State Action, which is conduct by an officer that bears a sufficiently close nexus to a state so that the action is treated as though it is by the state. -West's Encyclopedia of American Law, edition 2
Will Big Brother install new signs for a 65 MPH speed limit on the Dragon, which would lower the death rate by 400% according to TDOT? Of course not...there's too much profit to be made robbing the sheeple. Cops on the Dragon have grown accustomed to their $100,000 annual paychecks, and judges have grown accustomed to their $150,000 paychecks.
COP. 2. to steal; filch. 3. to buy (narcotics). 4. cop out, a. to avoid one's responsibility, the fulfillment of a promise, etc.; renege; back out. 5. cop a plea, a. to plead guilty or confess in return for receiving a lighter sentence. b. to plead guilty to a lesser charge; plea-bargain." -Random House Unabridged Dictionary
Not all cops want a Police State at Deals Gap. Pete Leary, the original owner of the Crossroads of Time Motorcycle Resort, was a retired cop who literally fought hard for biker rights against the corrupt sheriffs of North Carolina. Pistol Pete -- survivor of a gunfight at the CROT Corral -- won $70,000 from the Graham County NC sheriff for breaking Pete's arm during Pete's false arrest at CROT for standing up for tourists at the BMW Biker Gang Rally at Fontana Resort. Bootlegger sheriff Melvin Howell later was arrested for rape and blew his own head off -- or was "suicided". The current owner of CROT is Brad Talbott of Blount County TN, who may be related to the Talbott deputies in BCSO. Several biker cops warned bikers about the current Police State on the Dragon, and were punished by their commanders (see below). Suicidal sheriff Howell's chief deputy, Jerry Crisp, is apparently related to the BC "county executive" (mayor) William Crisp, named on the TDOT letter that illegally lowered the speed limit on the Dragon to 30 MPH in 2002. It was Graham County NC deputy Jerry Crisp who arrested ex-cop Pete Leary, owner of CROT, breaking his arm, who was fired for his false-arrest tendencies outside his jurisdiction (CROT is located in Swain County), but reemployed by Tennessee Valley Authority (TVA) police. TVA was also part of the renegade anti-motorcycle "Task Force" at the BMW Rally at Fontana. Here's how The Dragonater beat THP's #1 Top Speedgun trooper, and how you can win too, if you follow these steps in your trial for "speeding" on the Dragon: Video of Traffic Stop for alleged 60 MPH in a 30 zone - Rule #1: Never talk to a cop without your own audio recorder lie detector -- videocam is 100 times better THE DRAGONATER WINS IN TRAFFIC COURT AT DEALS GAP, RAISES SPEED LIMIT TO 65 MPH ON THE DRAGON - NOLLE PROSEQUI BY BLOUNT COUNTY ATTORNEY GENERAL. NO TESTIMONY, HEARING NOR TRIAL WHATSOEVER. 60 MPH SPEEDING TICKET DISMISSED WITH PREJUDICE, COSTS PAID BY THE STATE, IN BLOUNT COUNTY GENERAL SESSIONS COURT WITH JUDGE BREWER. THP TROOPER RANDALL HUCKEBY ADMITTED ON VIDEOTAPE DURING TRAFFIC STOP THAT ALL SPEEDING TICKETS NORTHBOUND ON US129 AT MILE MARKER 0.5 ARE FEDERAL JURISDICTION, NOT STATE JURISDICTION (VIDEO BY THE DRAGONATER). TDOT ADMITTED IN WRITING THAT THE MANDATORY TRAFFIC ENGINEERING SURVEY SPEED AUDIT WAS NEVER PERFORMED, IN VIOLATION OF TN CODE, THUS THE POSTED 30 MPH SPEED LIMIT ON THE DRAGON REVERTS TO THE DEFAULT 65 MPH IN TN CODE. THE DRAGONATER ALSO MADE VIDEO OF TROOPER HUCKEBY SPEEDING UP TO 60 MPH ON THE DRAGON IN A 30 MPH ZONE, WITHOUT MANDATORY EMERGENCY LIGHTS NOR SIREN, IN VIOLATION OF TN CODE, AND PERJURY IN HIS PERSONNEL FILE, WHICH SHOWED HIS $100,000+ SALARY. 2007 TDOT SAFETY AUDIT REPORT CONFESSED THAT THP'S JOB IS TO BAN ALL COMMERCIAL BUSINESSES ON THE DRAGON, SO THP TICKETS INCREASED 11,400% IN BLOUNT COUNTY. THP'S STALKER RADAR OPERATOR MANUAL CONFESSED THAT RADAR CANNOT MEASURE THE SPEED OF VEHICLES WITHIN 18 MPH OF ACTUAL SPEED. WATCH THIS SPACE FOR FULL EVIDENCE FILE. UPDATE 7 MARCH 2011 List of Court Documents in State of Tennessee vs John Lee, Docket Case Number T0061165, T844442, filed in Blount County General Sessions Court:
UNIFORM CITATION COMPLAINT - Signed "Under Protest TCA 47-1-308" DEFENSE MOTION #1 MOTION TO SUPPRESS STATEMENT DEFENSE MOTION #2 MOTION TO SUPPRESS RADAR TEST DEFENSE MOTION #3 MOTION TO DISMISS FOR LACK OF CORROBORATION OF CONFESSION DEFENSE MOTION #4 MOTION AND BRIEF FOR ORDER DIRECTING THAT THE STATE REVIEW FILES FOR BRADY MATERIAL DEFENSE MOTION #5 MOTION FOR JENCKS MATERIAL BRIEF IN SUPPORT OF DEFENDANT’S MOTION FOR THE PRODUCTION OF JENCKS MATERIAL IN GENERAL SESSIONS COURT DEFENSE MOTION #6 MOTION FOR AUDIO RECORDINGS DEFENSE MOTION #7 MOTION FOR SUBPOENAS BY PRIVATE PROCESS SERVER AND/OR MOTION FOR BENCH WARRANT DEFENSE EX PARTE MOTION TO PROCEED IN FORMA PAUPERIS SUBPOENA DUCES TECUM - Sgt Randall Huckeby, Badge No. 181, Tennessee Highway Patrol, District 1, 7601 Kingston Pike Knoxville, TN 37919. Two weeks return of service of process by Knox County Sheriff Office via Blount County Court Clerk, failed to meet deadline for hearing on 7 February 2001. Blount County Court Clerk Tom Hatcher illegally refused to issue subpoena to pro se defendant for private process server (felony official oppression). SUBPOENA DUCES TECUM - Sgt Randall Huckeby, Badge No. 181, Tennessee Highway Patrol, District 1, 7601 Kingston Pike Knoxville, TN 37919. Two days return of service of process by private process server met deadline for hearing on 7 March 2001. Blount County Court Clerk Tom Hatcher illegally refused to issue subpoena to pro se defendant for private process server (felony official oppression). Pro se defendant took the subpoena copy for Knox County sheriff and served it by mail via private process server (family member). SUBPOENA DUCES TECUM - Nathan Vatter, Traffic Engineer Region 1, Tennessee Department of Transportation, 7345 Region Lane, Knoxville, TN 37914; 865-594-2493. Two days return of service of process by private process server met deadline for hearing on 7 March 2001. Blount County Court Clerk Tom Hatcher illegally refused to issue subpoena to pro se defendant for private process server (felony official oppression). Pro se defendant took the subpoena copy for Knox County sheriff and served it by mail via private process server (family member). TRANSCRIPT: INITIAL APPEARANCE ARRAIGNMENT, by Miller & Miller Court Reporters (bypassing the official audio recording that often "disappears" when you need it for winning an appeal) DEFENSE MOTION FOR EXPUNGEMENT - Removes all court records of arrest for alleged speeding ORDER FOR EXPUNGEMENT - Download from Tennessee Administrative Office of the Courts DEFENSE MOTION # 8 FOR BILL OF COSTS AND BILL OF COSTS - Motion for Plainitff State of Tennessee to pay Defendant $450 for subpoenas, court reporter, transcript and postage. DEFENSE MOTION TO RE TAX COSTS - Download from Tennessee Administrative Office of the Courts DEFENSE NOTICE TO RE TAX COSTS - Download from Tennessee Administrative Office of the Courts ORDER TO RE TAX COSTS - Download from Tennessee Administrative Office of the Courts
Order of Expungment signed by Judge Brewer All motions, memorandums of law and subpoenas were drafted by The Dragonater, with subpoena service by private process server. All TN Open Records Act Requests at THP and TDOT were drafted and inspected by The Dragonater. Since the case was dismissed before any hearing took place, there are no memorandums of law regarding the theory of the case, no exhibits entered into evidence, no transcript of courtroom testimony, and no court orders regarding the law and facts of the case. There will probably be an expungement of the existing court record. Since there was no appeal of a jury verdict, there is no legal precedent to cite in future cases. But this same legal defense can be used in any future case to win dismissal of traffic tickets on the Dragon. It's impossible for TDOT to conduct a Traffic Engineering Survey Speed Audit until speed limit signs are changed to 65 MPH, and all cops are eliminated from the Dragon. Trial Notebook:
1. TN Code is posted on this webpage (used by attorney "Mr X" to negotiate dismissal for failure to prosecute by omission of Essential Elements). 2. Case citations: CITY OF OAK RIDGE v. DIANA RUTH BROWN, COURT OF APPEALS OF TENNESSEE AT KNOXVILLE, No. E2008-02219-COA-R3-CV, MAY 8, 2009. Requires Traffic Engineering Survey Speed Audit before lowing speed limits in Tennessee. 3. Video of traffic stop - Sgt Huckeby admitting Federal jurisdiction and felony Official Oppression, perjury by trooper about running Stalker radar, destroys his credibility as a witness) 4. Video of Sgt Huckeby speeding on the Dragon - Constitutional Equal Protection defense, proof of perjury by Huckeby in his annual review in his THP personnel file, destroys his credibility as a witness. 5. TDOT Safety Audit Review on the Dragon (starring The Dragonater, admitting an unconstitutional Police State that quadrupled the death rate on the Dragon) 6. TDOT letter from attorney Marion Hilt admitting a Speed Audit was never done on the Dragon before lowing speed limit to 30 mph, 40 mph nor 55 mph, so the default speed limit is 65 mph. 7. TDOT letter from Nashville illegally ordering speed limit reduced to 30 mph on the Dragon in 2002. 8. TDOT work order for signs reducing speed limit to 30 mph in 2002. 9. Sgt Huckeby's personnel file from THP HQ in Nashville, proving he can't guess speed within 4 MPH under ideal conditions, is a convicted deadbeat dad with three wives under financial pressure to sue more traffic tickets 10. THP website bragging that THP increased tickets 11,400% for motorcyclists in Blount County, while reducing tickets 30% for all other drivers 11. Stalker radar operator's manual used by THP, admitting that radar cannot measure the speed of traffic within 18 MPH. 12. List of questions at Preliminary Examination - These exhibits from THP and TDOT were to be authenticated during testimony of subpoenaed witnesses. Strategy at Prelim Exam is the opposite of strategy at jury trial. Defense tactic at Prelim Exam is to set prosecution witnesses up for perjury at jury trial; get discovery of evidence which is only allowed when Motions to Suppress are filed; get discovery of public records by subpoena decus tecum; discover theory of prosecution's case; psych out prosecution.
This exhibit list was used by attorney "Mr. X" to negotiate dismissal for failure to prosecute by omission of Essential Elements and Constitutional Equal Protection defense. These exhibits are not part of the court record in this speeding ticket case, since the Blount County attorney general dismissed the case before any court hearings. Burden of proof at Preliminary Examination is 50.01% -- a mere coin toss -- to see if the facts meet the test for probable cause (a mere allegation sworn by oath under penalty of perjury). But Essential Elements of law must always be 100%. General sessions court has Sucker Court via "trial by magistrate" that requires waiver of jury trial, or Real Court via Preliminary Examination. Judge Brewer illegally censored the words "jury trial" and "preliminary examination" from all defendants during the Initial Appearance arraignment. The Dragonater was the only defendant to demand prelim exam, execpt perhaps a DUI defendant represented by an attorney who was also the only other defendant to hire a private court reporter. The Law of the Case means that if a defendant does not specifically name a law, rule, constitution or case citation, they don't exist in that one case. Judge Judy is for suckers...that's why the pathological liars in the media mafiya pay her $30-million every year. A judge is not your friend. A prosecutor is not your friend. A cop is not your friend. They are your sworn enemy. Bibliography of strategy and tactics used for winning this case:
Bible for criminal defense is Tennessee Criminal Trial Practice, by Thomson West. Beat Your Ticket Go To Court And Win, by the lawyers at Nolo Press. Best pro se introduction to winning in any type of court. How to Beat a Speeding Ticket, free ebook by biker lawyer Norman Fernandez, covers topics censored by Nolo, such as how to win any photo radar ticket case by ignoring the ticket and refusing to appear in court, for lack of personal service of process. City of Cookeville v. Onks, 1993 WL 398472, Tenn.App., 1993, by ignoring 200 parking tickets and refusing to appear in court, for lack of personal service of process. Also covers Constitutional Equal Protection arguement at TN Supreme Court, proving that when govt employees get a right or immunity (free parking), then everyone gets that same right or immunity. Free video How Any Idiot Can Beat a RADAR Speeding Ticket, by Pastor Rick Strawcutter ND, certified paralegal, winner of several appeals citing The People vs Ferency re Essential Elements of radar cases, as seen on MSNBC's 50-Caliber Militia. National Motorists Association's Guerilla Ticket Fighter CD and Ticket Defense Kit. NMA are the nice folks who got the insane 55 MPH national speed limit reversed, and who will pay your speeding ticket if you lose in court.
Speeding is a Class C Misdemeanor in TN Code, punished by up to 30 days in jail, so defendants have a right to an attorney, and use Tennessee Rules of Criminal Procedure (TRCrimP). Municipal ordinance tickets in city court are civil cases using Tennessee Rules of Civil Procedure, (TRCP) without a right to an attorney. Service of subpoenas and collection of judgments use TRCP in criminal cases. TN Rules of Evidence apply in all courts. TN Open Records Act Requests by The Dragonater got the Top Secret TDOT report banning business on the Dragon, THP trooper personnel files with $100,000 salary records, and THP's Stalker radar operator manual admitting radar does not work.
"The Tennessee Highway Patrol (THP) has decided to take action in eliminating commercial activity along state right-of-way in the project area. This enforcement is intended to disallow persons from taking photographs of vehicles and motorcyclists with the intent of selling the photographs at a later date. The THP contends this activity is a hazard o public safety. It is recommended that signs be placed at the entrance of each end of the route advising this policy." -Tennessee Department of Transportation, ROAD SAFETY AUDIT REVIEW, STATE ROUTE 115 (U.S. 129), BLOUNT COUNTY, 25 July 2007 (starring The Dragonater -- but TDOT and foreign contractors can have as many traffic scameras as they want to fleece the sheeple)
THP increased biker tickets 11,400% on the Dragon
"Lastly, I want to give everyone a friendly warning. Theres a new Trooper thats been working the Dragon lately, and word around the mountain is that he used to be the #1 ticket writer in Knoxville. Well, it looks like hes trying to get his record back and I have heard of a few that have received tickets for 7-8 over the limit. Fair enough, since the speed limit is only 30 now. I do remember a day, not too long ago, when the limit was 55, then it was turned down to 40 and now in the last few years its become 30. This road can be safely navigated by most at a higher speed, and 40 would be a reasonable speed for all involved, but for whatever reason the powers that be have decided 30 was the best number, although it had been 55 for almost 50 years. Speeders will always speed, but controlled riders will always ride within their abilities and limits. 'nuff said." -DealsGap.com, 20 June 2009 Huckeby, Randall D Department: TN DEPT OF SAFETY AT DEALS GAP Class Title: THP SERGEANT ON THE DRAGON Current hourly rate: $28.89 FY 2007-08 salary: $60,084.00 Overtime dollars: $10,181.87 Total = $70,265.87 (not counting private part-time employment for "security", nor double-dipping military deployment as MSGT while on police salary, nor benefits) 1,500 tickets per year, #1 Ticket Writer in Tennessee? 11,400% increase in THP tickets on the Dragon List of the $100,000 salaries of Blount County Sheriff Office, not counting benefits and overtime, requiring a 21% tax increase in 2011 COP. 2. to steal; filch. 3. to buy (narcotics). 4. cop out, a. to avoid one's responsibility, the fulfillment of a promise, etc.; renege; back out. 5. cop a plea, a. to plead guilty or confess in return for receiving a lighter sentence. b. to plead guilty to a lesser charge; plea-bargain." -Random House Unabridged Dictionary "10% of cops are honest, 10% are dishonest, and 80% wish they were honest." -Detective Frank Serpico NYPD, Knapp Commission "The US Supreme Court said in Miranda that there's 40,000 police jurisdictions in America. We expect one civil rights crime in every one of those agencies every day. 40,000 times 365 days a year is 14,600,000 crimes committed by police every year. Hell, there's only 14,200,000 crimes committed by the criminals. The police commit more crimes against the People than the criminals commit!" -George Gordon Radio Show, The Policeman Is Not Your Friend, He Is Your Adversary, 2007 “I’ll burn your house down, set your dog on fire and there won’t be a member of your family left, do you understand me? I won’t hire it done, I will do it myself! Do you understand me?” -Blount County sheriff James Berrong, United Stated Court of Appeals, Sixth Circuit, Nuchols v. Berrong, No. 04-5645, July 11, 2005 The Battle of Athens TN and Blount County
How to make a request under the TN Open Public Records Act. Accessing unredacted payroll records of government employees in Tennessee. Tennessee Attorney General Opinion No. 01-155 Expunction of Records after Entry of Nolle Prosequi - Yes. Tenn. Code Ann. § 40-32-101(3) specifically requires a court to expunge public records upon the petition of a defendant who has had a nolle prosequi entered in his case. A court may not refuse or delay the expunction of public records after it receives a petition from a defendant who has had a nolle prosequi entered in his case. Under Tenn. Code Ann. § 40-32-101(a)(1), the trial judge has no discretion in granting the defendant’s petition for expunction. “Public records,” however, as defined by the statute, do not include arrest histories. Blount County Clerk refuses to issue subpoena in speeding ticket case, Dragonater issues arrest warrant for Blount County Clerk of Court TN Dept of Pre Crime arrests the Dragonater for speeding on the Dragon How to raise the speed limit on the Dragon THP closes US129 for 6 months after deadly crash East Tn Biker Cop Forum hacked, censored and banned The Dragonator, and emails from other members intercepted by police, for DARE to WIN in traffic court and raise the speed limit to 65 MPH on the Dragon. The Dragonater was permanently banned from East TN Riders forum by a police officer from Alabama.
Establishing Safe and Realistic Speed Limits
"Posting a lower speed limit does not decrease motorists speeds. Motorists drive at what is comfortable to them on that road. Lowering speed limits can cause an increase in crashes, and an increased speed limit can cause a decrease in crashes." -Michigan State Police
by Lisa M. Hoffmann Iron Mountain Daily News September 28, 2010 QUINNESEC, MICH. - State Police provided area residents a lot of information on how speed studies are conducted during an "Establishing Safe and Realistic Speed Limits" presentation at the Breitung Township Hall on Monday. Sgt. John Bruno of the Traffic Services Section of the Michigan State Police travels the Upper Peninsula with the Michigan Department of Transportation (MDOT) and local road commissions to establish safe and realistic speed limits. Monday's meeting was set to explain how roadway speeds are established. Approximately 15 area residents attended the meeting. He said the goal is to reduce the number of fatal and serious injury crashes, and thus, make roads safer. The three types of speed zones are: (1) statutory, which is set by the legislature. (2) absolute, which is 55 mph if not posted. (3) prima facia, which covers areas such as parks and school zones from 25, 35 and 45 mph. "If there are less than 30 access points (driveways, roadways), we can't put up prima facia," Bruno said. "We have to do a study." Although cities can set their own speed limits, townships cannot. "Posting a lower speed limit does not decrease motorists speeds. Motorists drive at what is comfortable to them on that road," Bruno said. "Lowering speed limits can cause an increase in crashes, and an increased speed limit can cause a decrease in crashes," he said. Bruno said in a study conducted in 22 states, there was no benefit to establish a speed limit below the 85th percentile of what motorists travel. "A black and white sign is not the solution to slow people down," he said. "If speed limits are posted properly, you need to pay attention because vehicles could be at that speed." Bruno addressed the history of U.S. 2 in Quinnesec. He said in 1942, the speed limit on the two-lane highway was 35 mph, and it was widened to five lanes in 1961. A traffic control order was established at 45 mph in 1962 and rescinded to 55 mph in 1970. Speed studies were conducted in 1979 and 1986 with a Traffic Safety and Engineer Study conducted in 1988 with no basis to lower the speed. Another speed study was conducted in 1996, and a stop signal was installed at U.S. 2 and Lake Antoine Road in 2001. "No further studies have been done since 2001," Bruno said. "There are 21 access points with seven of them abandoned or with little to no use. The other six are abandoned commercial drives." Bruno added there are 14 active access points in a half mile section of U.S. 2 in Quinnesec. The crash history indicates there were 21 crashes in eight years with one of those being a fatal. Bruno said 11,000 vehicles travel this roadway on any given day. "In eight years time, that is 32 million vehicles and 1.3 crashes per 1 million vehicle miles traveled," Bruno said. "Any crash that occurs is one too many." Bruno added this five-lane highway is a road that is used for motorists going from Point A to Point B. "If they see a black and white (speed limit) sign, that is not going to slow them down," he said. Residents of Lake Antoine Road in attendance at the meeting addressed the issue of log trucks going faster than 45 mph in front of their homes. A resident asked if there could be a different speed limit set for certain types of vehicles, such as a log trucks. "At 35 to 45 mph, my house is shaking like crazy," one resident said. "It is the vibration of speed, not a jake brake issue," another said. Bruno explained he has not seen two different speeds on the same road in the same area. "With two different speeds, there is more of a chance of crashes," he said. It was noted that any road not posted with a speed limit sign is 55 mph, but that does mean a police officer traveling by will not issue that driver a citation for traveling too fast for road conditions. MSP Establishing Realistic Speed Limits Booklet
People v Ferency, 133 Mich App 526; 351 NW2d 225 (1984) - Essential elements for conviction for speeding by use of police radar: 1. The officer operating the device has adequate training and experience in its operation. 2. The radar device was in proper working condition and properly installed in the patrol vehicle at the time of the issuance of the citation. 3. The radar device was used in an area where road conditions are such that there is a minimum possibility of distortion (spurious readings). 4. The input speed of the patrol vehicle was verified. This also means that the speedometer of the patrol vehicle was independently calibrated. 5. The speedmeter (radar) is retested at the end of the shift in the same manner that it was tested prior to the shift and that the speedmeter (radar) be serviced by the manufacturer or other professional as recommended. 6. The radar operator is able to establish that the target vehicle was within the operational area of the beam at the time the reading was displayed. 7. The particular unit has been certified for use by an agency with some demonstrable expertise in the area.
Michigan is home for Pastor Rick Strawcutter, certified paralegal, host of How Any Idiot Can Beat a RADAR Speeding Ticket:
§5.05 Engineering and Traffic Survey by Judicial Council of California www2.courtinfo.ca.gov January 1, 2011 When the officer’s means of determining the defendant’s speed included the use of radar or another electronic device, the prosecution must produce a current engineering and traffic survey to meet its burden of establishing a prima facie case that there was no speed trap. Without the survey, the arresting officer is incompetent to testify about the speed of the defendant’s vehicle even if the officer’s testimony is confined to his or her visual determination of speed. [See VC §40804(a); People v Conzelman (1994) 33 CA4th Supp 6, 8–9.] If the prosecution fails to justify the posted speed limit with such a survey, you must exclude all evidence of the defendant’s speed. [People v Studley (1996) 44 CA4th Supp 1, 3.] When an engineering and traffic survey is introduced into evidence, you must determine whether the survey: Complies with methods determined by the Department of Transportation; Considered prevailing speeds, accident records, and conditions not readily apparent to drivers (it also could have considered residential density and the safety of pedestrians and bicyclists) [VC §627; People v Goulet (1992) 13 CA4th Supp 1, 9–10]; and Justifies the speed limit—i.e., shows that the speed limit is just and based on sufficient lawful reason [13 CA4th Supp at 9]. The speed limit should be set at the nearest 5-mph increment to the 85th percentile critical speed unless the survey lists other factors justifying a reduction [Cal Manual on Uniform Traffic Control Devices (MUTCD) §2B.13].
85th percentile critical speed: The speed at or below which 85 percent of the traffic is moving. [See People v Goulet (1992) 13 CA4th Supp 1, 10.]
If you determine that the speed limit is not justified by the facts stated in the survey, you must dismiss the speeding charge. If you determine that the speed limit is justified by the survey, then you must decide whether guilt is proved beyond a reasonable doubt. [13 CA4th Supp at 13.] An engineering survey is not required:
For a local street or road [VC §40802(b)(1)], For a posted school zone while children are present [VC §40802(b)(2)], For a senior center zone [VC §22352(a)(2)(C)], For highways with statutory maximum speed limits [see, e.g., VC §22349(a)­­–(b) (exceeding 65 mph or 55 mph)], or When the officer is pacing with a speedometer or not using radar or lidar [see People v Goulet, supra, 13 CA4th at 3].
California Vehicle Code 22349. (a) Except as provided in Section 22356, no person may drive a vehicle upon a highway at a speed greater than 65 miles per hour. (b) Notwithstanding any other provision of law, no person may drive a vehicle upon a two-lane, undivided highway at a speed greater than 55 miles per hour unless that highway, or portion thereof, has been posted for a higher speed by the Department of Transportation or appropriate local agency upon the basis of an engineering and traffic survey. For purposes of this subdivision, the following apply: (1) A two-lane, undivided highway is a highway with not more than one through lane of travel in each direction. (2) Passing lanes may not be considered when determining the number of through lanes. (c) It is the intent of the Legislature that there be reasonable signing on affected two-lane, undivided highways described in subdivision (b) in continuing the 55 miles-per-hour speed limit, including placing signs at county boundaries to the extent possible, and at other appropriate locations.
TN Code 39-16-516. Traffic offense citation quotas — Performance standards.
(a) A political subdivision or any agency of this state may not establish or maintain, formally or informally, a plan to evaluate, promote, compensate, or discipline a law enforcement officer solely by the issuance of a predetermined or specified number of any type or combination of types of traffic citations. (b) A political subdivision or any agency of this state may not require or suggest to a law enforcement officer that the law enforcement officer is required or expected to issue a predetermined or specified number of any type or combination of types of traffic citations within a specified period. (c) Nothing in this section shall prohibit a municipal corporation, a political subdivision or any agency of this state, from establishing performance standards for law enforcement officers that include issuance of traffic citations, but do not require issuance of a predetermined or specified number or any type or combination of types of citations as the sole means of meeting such performance standards. (d) As used in this section: (1) “Conviction” means the rendition of an order by a court imposing a punishment of incarceration or a fine; and (2) “Traffic offense” means an offense under title 55.
TN Code 39-16-403. Official oppression. —
(a) A public servant acting under color of office or employment commits an offense who: (1) Intentionally subjects another to mistreatment or to arrest, detention, stop, frisk, halt, search, seizure, dispossession, assessment or lien when the public servant knows the conduct is unlawful; or (2) Intentionally denies or impedes another in the exercise or enjoyment of any right, privilege, power or immunity, when the public servant knows the conduct is unlawful. (b) For purposes of this section, a public servant acts under color of office or employment if the public servant acts, or purports to act, in an official capacity or takes advantage of the actual or purported capacity. (c) An offense under this section is a Class E felony. (d) Charges for official oppression may be brought only by indictment, presentment or criminal information; provided, that nothing in this section shall deny a person from pursuing other criminal charges by affidavit of complaint.
TN Code 39-16-406. Suspension, removal and discharge from office.
(a) A public servant convicted under § 39-16-402, § 39-16-403 or § 39-16-404 shall be removed from office or discharged from the position. (b) A public servant elected or appointed for a specified term shall be: (1) Suspended without pay immediately upon conviction in the trial court through the final disposition of the case; (2) Removed from office for the duration of the term during which the conviction occurred if the conviction becomes final; and (3) Barred from holding any appointed or elected office for ten (10) years from the date the conviction becomes final. (c) A public servant who serves at-will shall be discharged upon conviction in the trial court. Subsequent public service shall rest with the hiring or appointing authority, provided that the authority has been fully informed of the conviction.
TN Code 55-8-103. Required obedience to traffic laws — Penalty.
It is unlawful and, unless otherwise declared in this chapter and chapter 10, parts 1-5 of this title with respect to particular offenses, it is a Class C misdemeanor, for any person to do any act forbidden or fail to perform any act required in this chapter and chapter 10 of this title.
TN Code 55-8-106. Public officers and employees — Exceptions.
The provisions of this chapter and chapter 10, parts 1-5 of this title applicable to drivers of vehicles upon the highways shall apply to the drivers of all vehicles owned or operated by the United States, this state or any county, city, town, district or any other political subdivision of the state, subject to specific exceptions as are set forth in this chapter and chapter 10, parts 1-5 of this title.
TN Code 55-8-108. Authorized emergency vehicles.
(a) The driver of an authorized emergency vehicle, when responding to an emergency call, or when in the pursuit of an actual or suspected violator of the law, or when responding to but not upon returning from a fire alarm, may exercise the privileges set forth in this section, but subject to the conditions stated in this section. (b) (1) A driver of an authorized emergency vehicle operating the vehicle in accordance with subsection (a) may: (A) Park or stand, notwithstanding other provisions of this chapter that regulate parking or standing; (B) Proceed past a red or stop signal or stop sign, but only after slowing down as may be necessary for safe operation; (C) Exceed the speed limits so long as life or property is not thereby endangered; and (D) Disregard regulations governing direction of movement or turning in specified directions. (2) Subdivision (b)(1) shall not relieve the driver of an authorized emergency vehicle from the duty to drive with due regard for the safety of all persons, nor shall subdivision (b)(1) protect the driver from the consequences of the driver's own reckless disregard for the safety of others. (c) (1) The exemptions granted under subsection (b) to a driver of an authorized emergency vehicle shall only apply when the vehicle is making use of audible and visual signals meeting the requirements of the applicable laws of this state, except that while parked or standing, an authorized emergency vehicle shall only be required to make use of visual signals meeting the requirements of the applicable laws of this state.
TN Code 55-8-152 - Speed limits - penalties
(a) Except as provided in subsection (c), it is unlawful for any person to operate or drive a motor vehicle upon any highway or public road of this state in excess of sixty-five miles per hour (65 mph). (d) (1) (A) Except as provided for certain counties in subdivision (d)(2), counties and municipalities are authorized to establish special speed limits upon any highway or public road of this state within their jurisdiction, except at school entrances and exits to and from controlled access highways on the system of state highways, which is adjacent to school grounds that are devoted primarily to normal school day activity. Such speed limit shall be enacted based on an engineering investigation, shall not be less than fifteen miles per hour (15 mph) and shall be in effect only when proper signs are posted with a warning flasher or flashers in operation and only while children are actually present. (B) In any county or municipality where the local legislative body does not establish special speed limits as provided for above, any person who shall drive at a speed exceeding fifteen miles per hour (15 mph) when passing a school during a recess period when a warning flasher or flashers are in operation, or during a period of ninety (90) minutes before the opening hour of a school or a period of ninety (90) minutes after the closing hour of a school, while children are actually going to or leaving school, shall be prima facie guilty of reckless driving. (f) (1) (A) Notwithstanding any provision of this section to the contrary, the department is authorized to lower the speed limits prescribed in this section, and on the state system of roads and highways, as it deems appropriate due to concerns regarding the roadway, traffic, or other conditions. Such authorization to reduce the speed limits set by this section shall be in addition to the authority conveyed by § 55-8-153. (B) When the department shall determine that it is necessary to reduce the speed limits set in subsection (a), the commissioner shall so indicate the reduced speed limit via a letter of policy statement, and the commissioner shall cause signs indicating the new speed limit to be erected. (C) Notwithstanding any other provision of this section to the contrary, the municipalities of the state are hereby authorized to set speed limits on the public roads and streets within their jurisdiction and not a part of the interstate and national defense highway system nor any access controlled highway on the state road and highway system. In addition, the counties of this state are hereby authorized to set speed limits on the public roads and highways within their jurisdiction and not a part of the interstate or state highway system. The speed limits for both municipalities and counties shall not exceed fifty-five miles per hour (55 mph). (2) Notwithstanding any provision of the law to the contrary, during the period in which this subsection (f) is in effect, any person who is arrested or receives a traffic citation for driving or operating a motor vehicle in excess of fifty-five miles per hour (55 mph) but less than seventy-five miles per hour (75 mph) on a highway of the interstate and defense highway system or a four-lane controlled-access highway which are federal or state highways, or in excess of fifty-five miles per hour (55 mph) or less than sixty-five miles per hour (65 mph) on a highway or road which has an existing speed limit of sixty-five miles per hour (65 mph) as of March 1, 1974, shall be charged with speeding and upon conviction shall not be fined more than the maximum fine nor less than the minimum fine for speeding as provided by law for such violation, nor shall any costs be imposed or assessed against such person. Costs shall be imposed in such cases should such person fail to appear or answer the traffic citation as required by law. Such conviction shall not be reported to the department of safety under the provisions of §§ 55-10-306 and 55-12-115. Such person shall not be required to attend driver education course as provided in § 55-10-301. Such conviction for speeding shall not result in suspension or revocation of operator's or chauffeur's license unless the excess speed is such as to constitute reckless driving, as set out herein. This subsection (f) shall not apply to trucks as defined in subsection (b) when traveling in excess of sixty-five miles per hour (65 mph) on all highways of the interstate and defense highway system and four-laned controlled-access highways, which are federal or state routes of this state or when traveling in excess of fifty-five miles per hour (55 mph) on any other highways of this state. A violation of this subsection (f) is a Class C misdemeanor. However, notwithstanding any provision of the law to the contrary, a violation of the reduced speed limits set by the department of transportation, pursuant to § 55-8-153, is a Class B misdemeanor, punishable by fine only, when employees of the department or construction workers are present. The amount of the fine imposed pursuant to § 55-8-153 shall not be less than two hundred fifty dollars ($250) nor more than five hundred dollars ($500). Notwithstanding any provision of this subsection (f) to the contrary, no provision of this subsection (f), nor of § 55-8-153, shall be construed so as to prevent the entry of a suspended sentence upon the conviction of a defendant for the first violation of the enhanced penalties provided for when the violation occurs within a work zone and when employees of the department of transportation or construction workers are present and when the trier of fact determines that extraordinary circumstances lead to the violation.
TN Code 55-8-153 Establishment of Speed Zones
(a) The department of transportation is empowered to lower the speed limits prescribed in § 55-8-152 in business, urban or residential districts, or at any congested area, dangerous intersection or whenever and wherever the department shall determine, upon the basis of an engineering and traffic investigation, that the public safety requires a lower speed limit. (b) Appropriate signs giving notice thereof shall be erected by the department at such places or put on the highway where the prescribed speed limits are effective. (c) (1) (A) The legislative authorities of municipalities shall possess the power to prescribe lower speed limits on highways designated as state highways in their respective jurisdictions when, on the basis of an engineering and traffic investigation, it is shown that the public safety requires a lower speed limit. (B) Engineering and traffic investigations used to establish special speed zone locations and speed limits by municipalities on state highways shall be made in accordance with established traffic engineering practices and in a manner that conforms to the Tennessee manual on uniform traffic control devices (MUTCD). The investigations shall be documented and documentation shall be maintained by the jurisdiction performing or sponsoring the investigation. (C) All signs, signals and other forms of public notification of the speed limits, road hazards and other traffic conditions shall comply with the MUTCD. (2) The legislative bodies of municipalities shall also possess the power to prescribe lower speed limits within certain areas or zones, or on designated highways, avenues or streets that are not designated as state highways in their respective jurisdictions, and to erect appropriate signs and traffic signals. (d) The legislative body of any county, except the legislative bodies of any counties having a commission form of government, has the power to prescribe such lower speed limits as it may deem appropriate on any road being maintained by such county and shall erect appropriate signs and traffic signals. In those counties having a commission form of government, the board of commissioners has the power prescribed in this section.

Tennessee Highway Patrol Ticket Quota Uncovered TheNewspaper.com 7/10/2009 Tennessee Highway Patrol memo establishes numeric traffic ticket quota required for earning promotions and extra pay. Tennessee Highway Patrol (THP) troopers are punished for failing to issue a specific number of speeding tickets in at least one part of the state. Attorney Fletcher Long provided WTVF-TV with a copy of what THP Sergeant Clifford M. Babits posted on the wall of the Troop C station in Robertson County. "I can no longer justify fives on evaluations for troopers not producing activity," Babits wrote. "I require three things. 1. Answer the radio, 2. Work your crashes, and 3. WRITE TICKETS. I take some of the blame for not properly motivating ya'll in the area of activity. Overall activity last year (2008) was well below the district average... Because activity plays such a high part of an everyday road trooper's requirement, it is going to weigh heavy on yearly evaluation scores." These scores, with 'five' being the highest rating, are key to winning promotions, extra pay and the most desirable types of assignments. According to the memo, scoring is based solely on the number of tickets issued, although other factors such as routinely failing to follow orders can result in a lowered score. Babits set six hundred tickets a year, or three tickets per day, as the bare minimum. "Let me stress I am not putting a quota on anyone," Babits wrote. "I don't care if a trooper writes below the 600 mark, it is his or her evaluation score, not mine. If a trooper turns in 600 citations per year, his or her overall evaluation will not be above the average score of three... I must be able to justify giving a trooper a five. Low activity is a killer." It takes 800 citations per year, or four tickets per day, to earn the top score. A rating of four is earned by writing 700 tickets per year or 3.5 per day. Those who fail to exceed the average score of three are punished by not being allowed to earn time-and-a-half pay on overtime assignments. "Effective immediately, a trooper that does not produce above average activity (17.5 tickets per week) will not be able to work grant overtime in my county," Babits wrote. Tennessee Highway Patrol officials deny any quota exists. Many state police forces use the average number of tickets written by troops or stations to encourage a steady year-on-year increase in the number of speeding tickets issued. A copy of the memo is available in a 75k PDF file at the source link below. Source: PDF File Troop C Memo (Tennessee Highway Patrol, 7/10/2009)
Tennessee SB 2703 by *Burchett, Burks, Black, Ford, O.. (HB 2952 by *Brooks H, Hardaway, Haynes, Faulkner.) Local Government, General - As enacted, prohibits local governments from punishing or rewarding law enforcement officers solely based on the number of traffic citations issued or collected upon. - Amends TCA Title 39, Chapter 16 and Title 55, Chapter 8. Fiscal Summary Bill Summary This bill prohibits a political subdivision or any agency of this state from establishing or maintaining, formally or informally, a plan to evaluate, promote, compensate or discipline a law enforcement officer according to the officer's issuance of a predetermined or specified number of traffic citations. This bill further prohibits a political subdivision or any agency of this state from requiring or suggesting to a law enforcement officer that the officer is required or expected to issue a predetermined or specified number of traffic citations within a specified period. Under state statutory law, any person holding a state, county, or municipal office is subject to removal from such office for any of the following, unless the removal of such person is otherwise provided for: (1) Knowingly committing misconduct in office; (2) Knowingly neglecting to perform any duty mandated by any of the laws of the state; (3) Voluntary public intoxication; (4) Any form of gambling; or (5) Any act constituting a violation of any penal statute involving moral turpitude. This bill specifies that any elected official who violates its provisions commits an offense for which such official could be removed from office as discussed above. If the person who violates the provisions of this bill is not an elected official, then such person would be subject to disciplinary action, including removal from such person's position. This bill specifies that it does not prohibit any political subdivision or agency of the state from establishing performance standards for law enforcement officers that include issuance of traffic citations, but do not require issuance of a predetermined or specified number or any type or combination of types of citations as the sole means of meeting such performance standards. ON MAY 10, 2010, THE SENATE SUBSTITUTED HOUSE BILL 2952 FOR SENATE BILL 2703, ADOPTED AMENDMENTS #1 AND #2, AND PASSED HOUSE BILL 2952, AS AMENDED. AMENDMENT #1 clarifies that this bill prohibits plans that discipline or reward an officer "solely by" issuance of a predetermined or specified number of any type or combination of types of traffic citations. AMENDMENT #2 removes this bill's provision whereby a violation of this bill would be considered misconduct in office under statutory and constitutional law, and would subject a person to removal from office. 05/07/2010 Signed by Governor Effective date(s) 07/01/10
Tennessee Pattern Jury Instructions T.P.I. -- CRIM. 25.02 OFFICIAL OPPRESSION Any public servant who commits the offense of official oppression is guilty of a crime. For you to find the defendant guilty of this offense, the state must have proven beyond a reasonable doubt the existence of the following essential elements:1 (1) that the defendant was a public servant acting under color of office or employment; and (2)(a) that the defendant intentionally subjected another to mistreatment or to arrest, detention, stop, frisk, halt, search, seizure, dispossession, assessment, or lien that the defendant knew was unlawful; or (b) that the defendant intentionally denied or impeded another in the exercise or enjoyment of any right, privilege, power or immunity when the defendant knew the conduct was unlawful. "Public servant" means a person elected, selected, appointed, employed, or otherwise designated as one (1) of the following even if the public servant has not yet qualified for office or assumed the duties: (A) an officer, employee, or agent of government; or (B) a juror or grand juror; or (C) an arbitrator, referee, or other person who is authorized by law or private written agreement to hear or determine a cause or controversy; or (D) an attorney at law or notary public when participating in performing a governmental function; or (E) a candidate for nomination or election to public office; or (F) a person who is performing a governmental function under claim of right although not legally qualified to do so.2 A public servant acts under color of office or employment if the servant acts or purports to act in an official capacity or takes advantage of such actual or purported capacity.3 "Intentionally" means that a person acts intentionally with respect to the nature of the conduct or to a result of the conduct when it is the person's conscious objective or desire to engage in the conduct or cause the result.4 FOOTNOTES 1. Tenn. Code Ann. § 39-16-403(a). 2. Tenn. Code Ann. § 39-16-403(b). 3. Tenn. Code Ann. § 39-16-401(3). 4. Tenn. Code Ann. § 39-11-106(a)(18). COMMENTS 1. Official oppression is a Class E felony. Tenn. Code Ann. § 39-16-403(c).
Ethics Provisions Provided by Statute The local attorney general or city attorney has a legal "duty" to investigate a written allegation that an officer has been guilty of any of the mentioned offenses. If he or she finds that "there is reasonable cause for such complaint, he shall forthwith institute proceedings in the Circuit, Chancery, or Criminal Court of the proper county." However, with respect to the city attorney, there may be an irreconcilable conflict between that duty and the city attorney's duties to the city, the mayor, and the rules of professional responsibility governing attorneys. Also, an attorney general or city attorney may act on his or her own initiative without a formal complaint (T.C.A. § 8-47-101-102). The officer must be removed from office if found guilty (T.C.A. § 8-47-120).
TN Code §47-1-308. Performance or acceptance under reservation of rights Title 47. Commercial Instruments And Transactions Chapter 1. Uniform Commercial Code - General Provisions (1) "A party who, with explicit reservation of rights, performs or promises performance or assents to performance in a manner demanded or offered by the other party does not thereby prejudice the rights reserved. Such words as 'without prejudice,' 'under protest' or the like are sufficient."

‘Official oppression’ difficult to prove Now that the Knox County Ethics Committee found County Mayor Mike Ragsdale and two of his top staffers engaged in “official oppression” by retaliating against the county’s internal auditor, officials will try to determine if they broke the statute in a criminal sense. The state law covering official oppression is a criminal statute, though it’s also one of the laws that’s used as part of the county’s ethics policy. If a jury in criminal court agrees there’s a violation of that law, jail time is a possibility. The Ethics Committee, operating under a lower standard of proof, found Ragsdale, Chief Financial Officer John Troyer and Chief Administrative Officer Dwight Van de Vate merely violated the ethics policy. Internal Auditor Richard Walls alleged that the mayor and his staff retaliated against him for a series of audits that found numerous violations of county policy and state law in the handling of finances. The Ethics Committee agreed and turned the matter over to District Attorney General Randy Nichols for a possible criminal investigation. According to Knox County Chief Deputy Law Director Joe Jarret, the statute addresses public servants, acting under the “color of law” or the authority of government, who use that authority through an illegal act to prohibit an individual from exercising legal rights. “It’s a very broad, vague law,” said criminal defense lawyer Gregory P. Isaacs. “The person violates the statute who was acting under color of the law, using their authority that was given to them in an unlawful manner. “Under this statute, you are not entitled as a citizen to go down and swear out a warrant and have somebody arrested. You have to go through a prosecutor. This is a crime that has to be taken to a grand jury, which is extremely rare.” The law does not, however, prevent a resident from taking out a warrant for other criminal charges, he said. Both Isaacs and Jarret said use of the law in Tennessee is very rare and typically is applied to law enforcement officers accused of overstepping their authority. “It would be easy to obtain an indictment; however, it would be very difficult to obtain a conviction,” Isaacs said. “The problem is it’s a Class E felony, which is one to two years in prison.” During the Ethics Committee hearings, officials said the law carries up to a six-year sentence, but Isaacs said six years would apply only if there were prior convictions. Proving the case would be a “very difficult proposition,” he said. “It looks like another political controversy that would be lingering around for many months if not longer,” Isaacs said. Jarret said his research found very few cases applying the law. Before its hearings last month, the Ethics Committee was given a copy of the statute and jury instructions, said Jarret, who served as counsel to the panel. The standard that members used to determine an ethics code violation was a “preponderance of the evidence.” A criminal proceeding would use the higher standard of evidence “beyond a reasonable doubt.” Isaacs said the law “protects police officers and elected officials from getting arrested” under that statue without an indictment. Isaacs questioned whether the district attorney general’s office would recuse itself from the case like it has in other cases involving county officials, including criminal probes of the mayor’s office that have not found criminal violations or didn’t have sufficient evidence to obtain a conviction. According to the statute, it’s a criminal act when “a public servant acting under color of office or employment … intentionally subjects another to mistreatment or to arrest, detention, stop, frisk, halt, search, seizure, dispossession, assessment or lien when the public servant knows the conduct is unlawful or intentionally denies or impedes another in the exercise or enjoyment of any right, privilege, power or immunity, when the public servant knows the conduct is unlawful.” The reference to “mistreatment” is the basis for Walls’ complaint, Jarret said. The Knox County Commission, as mandated by the state, adopted an ethics policy, which lists a number of state statutes that could be violations, including official oppression. “The cases I’ve found deal with law enforcement abusing their power of office, like a sheriff selling pardons,” Jarret said “That’s why it was difficult to advise the Ethics Committee. All I was able to do was show them examples where it was used, and I gave them a copy of standard jury instructions and a copy of the law itself.” Jarret said the rarity of the law’s use “makes it challenging, that’s for sure.” Richard Walls’ complaint against Knox County Mayor Mike Ragsdale and top staff Comments I believe that former Knoxville Mayor Victor Ashe was found guilty of political retaliation against the civil rights of several firefighters in the local federal court. I guess Ragsdale learned his politics of oppression from Ashe. Since the taxpayers of Knox Co. have paid these s*** to do their jobs (stealing ect.), they will claim they were doing their job and will be on Bill Locketts door demanding that the taxpayers now pay for their defense. They have no shame! I think they have another hospitality acct set up because rags is refusing to disclose who paid for his vacation to Kosovo, will only say that 5 businesses paid for it. Thank God little jack has turned loose some of the fine reporters that are at kns! I think Mr. Walls has sufficient evidence to back his complaint. The letters sent to the American Institute of Certified Public Accountants, the Tennessee Society of Certified Public Accountants and the Institute of Internal Auditors were crafted on official stationay from the Office of the Mayor of Knox County and states the purpose: "This letter is intended to serve as an official complaint against a member..." Signed by John Troyer, CPA in an official capacity as Chief Financial Officer and DVD in his official capacity of Chief Administration Officer. Very interesting. This is the same editor that convinced the publisher and Scripps in general to take his "opinions and editorials" into court and sue the Knox County Commission for violation of the Sunshine law. "Official oppression’ difficult to prove". Yes, especially if you don't try very hard.
VIDEO: Warning to Bikers at Deals Gap "EVERYONE WHO GOES TO DEALS GAP------PLEASE READ THIS-----I'M TRYING TO HELP YOU ALL OUT Hey, just to give you a heads up for next season----we have a new captain in the knoxville district which covers blount county------he has already told us that we will put a stop to the "race track driving" up on the dragon this year------so far what they have said is that the speed limit is 30mph and "ANYONE" caught going 50mph and up will be charged with reckless driving and will be jailed and their bike/car will be impounded-----I am telling you this because i am trying to put the word out to everyone i know----or to people who know people that go up there-----I don't want to have to take people to jail for this---however I have bills to pay and I kind of like having a job---so never the less I have to do what i am told to do-----so the way i see it is that if killboy.com knows and you know and dealsgap.com knows and whoever else can find out can pass this along---it will benefit everybody--------feel free to copy and past this and let everyone know-------if you get caught up there it is only your fault because i am trying to pass the word along for everybody's sake-----we are not just going to come up from the bottom like in years past---they are gonna have us go through cherohala skwy in north carolina and come from the top down to the bottom----so no more free rides on your way back to the state line----they are gonna bring unmarked cars with radio's and have unmarked bikes up there----this will be a joint effort with blount county and the THP-----The entire THP motor unit will be brought in 2 people at a time(for a week long assignment) to be paired up with the local car----they plan on busting tail at least 3-4 times right off the bat---then they will sit back and see what takes place-----these orders came from Nashville Headquarters---we have a complete brand new higher ranking structure from the very tip top spot down----they have said we will work until the problem is solved----just wanted to give you and your friends a heads up..........SIGNED..........Trp Cagle with THP assigned to Blount County Division----(don't come crying to me if you get busted----cause I'm telling you now)" -a state trooper, Myspace.com, ETR, Gap Paranoia, 01-31-2007
"You may recall a little letter that circulated last winter from a certain TNHP officer Cagle that warned of all this. That officer sacrificed his job basically to get the word out to us, and now works weight scale duty on I-40 as I understand it, as a result." -Killboy, ETR Forum, Crackdown hurting, August 5, 2007 "I've seen a lot of outrageous things happen in politics since I've been working here, but this tops them all. It's the equivalent of issuing a series of O.J. Simpson autographed steak knives. And no, this isn't a joke. Here's the new rule: Kill a biker, get a highway named after you. As a tourist who has spent a lot of family vacation dollars in Sen. Clabough's district, if those signs go up, you'll never see another penny of mine in Knox or Blount counties." -Greg Harrison, Vice President of Communications for American Motorcyclist Association, American Motorcyclist magazine, A Monumental Mistake, June 1999 “Please read the enclosure that presents the facts by someone involved in the investigation of this tragic accident. I grew up knowing Sen. Koella for the last 30 years. The bill has been passed and signed. There is NOTHING you can do to change that, and if you choose not to come to Tennessee, we will be better off without you.” -Senator Bill Clabough, American Motorcyclist Association, American Motorcyclist magazine, Greg Harrison Column: Contemptible, September 1999
Trucks on the Dragon Warning to Bikers at Deals Gap Leave Bikers Alone at Deals Gap! Aircraft Radar Speed Enforcement on the Dragon at Deals Gap Frustrated Driver Files Federal Lawsuit Against “Speed Trap”
Tail of the Dragon named Top Road in USA Citizen Times March 16th, 2011 ASHEVILLE, N.C. -- The Blue Ridge Parkway and U.S. 129, aka The Tail of the Dragon, have been named two of the top roads in the country for scenic motorcycle riding. Members of the American Motorcyclist Association voted on their favorite roads in America, and the top 15 selections represent scenic highways and byways throughout the country. “When it comes to the best places in the country to ride motorcycles, no group of riders is more knowledgeable than AMA members,” said AMA President and CEO Rob Dingman. “No group puts in more miles, samples more asphalt or rides more roads. “By publicizing our favorite roads in American Motorcyclist magazine, AMA members will seek out these destinations and bring tourism dollars into local communities through the purchase of food, fuel, lodging and more,” Dingman said. The editors of American Motorcyclist magazine used the 230,000-member, non-profit association’s website, AmericanMotorcyclist.com, to accept nominations and tally votes for members’ favorite roads. Nearly 100 roads made the ballot, and the magazine published the top 15 roads in the April issue as well as a list of “honorable mentions.” The top 15 routes in USA today: 1. Beartooth Highway, Montana and Wyoming. 2. Blue Ridge Parkway, North Carolina. 3. U.S. Route 129 — The Tail of the Dragon — on the North Carolina-Tennessee border. 4. U.S. Route 550, from Ouray to Durango, Colo. 5. California Route 1, Pacific Coast Highway. 6. Going to the Sun Road, Glacier National Park, Montana. 7. Cherohala Skyway, North Carolina and Tennessee. 8. California Route 36. 9. U.S. Route 12, Lolo Pass, Idaho and Montana. 10. Angeles Crest Highway, California Route 2. 11. Natchez Trace, from Natchez, Miss., to Nashville, Tenn. 12. U.S. Route 33, Harrisonburg, Va., to Seneca Rocks, W.Va. 13. California Route 58, McKittrick to Santa Margarita. 14. Ohio Route 170, Calcutta to Poland. 15. Washington Route 129 and Oregon Route 3, Clarkston, Wash., to Enterprise, Ore.
See also: THP Police State on the Dragon Prosecutors, judges, governors, legislators, TN Code say car insurance is never mandatory Trackback links Deals Gap is 65mph? R6 Forum - Sheriff Deputy SpidermanSS says "the R1 and R6 Forums have a convention there every year. This year in June will be our 10th year. Ride the dragon at a pace that you are comfortable with and don't push it to "keep up" with someone and you will have a great time ride there. I'd steer clear of the drama and net BS over that place there is also more incredible roads near there than just the gap." Speed Limit at the Dragon?? Triumph 675 Forum - "I read a chunk of the page and they passed a law in MI that is similar to this one. I have successfully fought a ticket and had it dismissed using a similar defense described in the article. In MI it is required to that a speed study be completed before setting a speed limit on any road except highways, school zones, residential and business districts. So when the law passed in 2006 90% of the speed limits are now illegal, and a little effort will get you out of the ticket. The cop cannot argue that you were traveling to fast for the conditions because there is no base line to set the speed off of." Note that it is impossible for TDOT to do a valid speed study until the speed limit is officially posted as 65 MPH, for at least 1 year, to give tourons, pirates and locals time to learn the "new" speed limit. Speed Limit at the Dragon?? NC Sportbikes Forum - "So, makes sense to me. Sounds good to me! Natural selection will remove those who can't control their bikes at 65. Sounds good to me. If I can just slow down about 10 mph then I'll be legal on the Dragon. Great video's and information... It's a must read before we head up there... And another reason why i'm so anti law enforcement (so few that are worth a damn...) even when they start out with good intentions they end up letting the power go to their heads.... Wish that wasn't the case." Deal's Gap - Speed Cameras - "If ticketted I will use this page as a framework for my defense....Technically, it states the legal limit on the Dragon is 65 MPH!!" US Route 129 AKA 'Tail of the Dragon' - "Can you survive 18 cops and $800 tickets per 11 miles? Perhaps you can when you read the case file that proved in court that the Real World speed limit is 65 mph on the Dragon, not the illegal posted 30 mph. Case dismissed, costs paid by the State." - John Lee, Dragonaters Blog
The following court order can be cited as an "advisory opinion" in any court in Tennessee: People v. Goulet, 13 Cal.App.4th Supp. 1 Appellate Department, Superior Court, Ventura County, California [Crim. A. No. 3227. Dec 18, 1992.] THE PEOPLE, Plaintiff and Respondent, v. JUDITH ANN GOULET, Defendant and Appellant. (Municipal Court of Ventura County, No. 91M112920, Kenneth W. Riley, Judge.) (Opinion by Osborne, P. J., with McNally and Steele, JJ., concurring.) COUNSEL Judith Ann Goulet, in pro. per., for Defendant and Appellant. Michael Bradbury, District Attorney, Kent Baker and William Redmond, Deputy District Attorneys, for Plaintiff and Respondent. OPINION OSBORNE, P. J. A. Introduction Several cases have clearly explained the speed trap laws adopted by the California Legislature, the policies behind them, the burden placed on the prosecution, and the general statutory requirements for an engineering and traffic survey to justify a speed zone reduced below 55 miles per hour. In this case, we apply those requirements and determine that a survey did not justify the speed zone adopted by a local authority. [1] The Legislature has declared a strong public policy against the use of speed traps. (People v. Halopoff (1976) 60 Cal.App.3d Supp. 1 [131 Cal.Rptr. 531].) The policy has been explained various ways. It furthers a policy of prevention by plain sight patrolling rather than punishment after the fact, and encourages observance of all the rules of the road. (Fleming v. Superior Court (1925) 196 Cal. 344, 349 [238 P. 88].) "Commentators have suggested that the Legislature was also motivated by a desire to eliminate clandestine methods of traffic enforcement designed to augment local revenues through exorbitant fines. [Citations.]" (People v. Sullivan (1991) 234 Cal.App.3d 56, 58 [285 Cal.Rptr. 553].) Speed trap rules are not applicable to evidence of speed based on use of a speedometer without any use of radar. The Legislature may anticipate that [13 Cal.App.4th Supp. 4] when an officer is driving a vehicle and enforcing prima facie speed laws by observing traffic, roadside conditions, his and/or her own perceptions of safety under the circumstances, and then noting speed from the speedometer of the patrol vehicle, the judgment is likely to be similar to the reasonable and prudent majority of drivers, and not be determined merely by a speed limit which, for political, revenue, or other reasons, a local authority may have set below what is reasonable and necessary for safe and efficient movement of traffic. Traffic rules account for most of the contact by average citizens with law enforcement and the courts. Enforcement of laws which are widely perceived as unreasonable and unfair generates disrespect and even contempt toward those who make and enforce those laws. Whatever the motivation, the Legislature has spoken clearly and emphatically about speed trap laws. B. Facts Appellant was cited for violation of section 22350 fn. 1 of the California Vehicle Code fn. 2 , exceeding the basic speed limit. The citation states her approximate speed as 52 miles per hour, the prima facie speed limit as 35 miles per hour, and the safe speed as 35 miles per hour. The citation notes the use of radar. Deputy Berg testified that he visually estimated appellant's speed at 50 to 55 miles per hour in a posted 35 miles per hour zone, the radar showed 52 miles per hour, he pursued and stopped her, and he cited her. fn. 3 He stated she was traveling near a senior citizens' complex where there is often a lot of foot traffic and bicyclists. In support of the 35 mph speed zone, an engineering and traffic survey was performed May 19, 1988. The road is a major artery with a raised median center divider, with two striped lanes in each direction. For traffic in the direction appellant was traveling, the 85th percentile critical speed was 48 mph. That is, 85 percent of the surveyed vehicles were traveling at a speed of 48 mph or less. The average speed was 43 mph. The speed limit of 35 mph was exceeded by 95 percent of the drivers. [13 Cal.App.4th Supp. 5] The "Accident and Roadside Features Review" section of the engineering and traffic survey lists "Unusual Roadside Features" as "1,000' radius curve, limited sight distance to commercial driveways." The segment length is 3,300 feet. The 1987 average daily traffic is stated as 12,300. The number of speed-related accidents (1985, 1986, 1987) is zero. fn. 4 C. Issues [2a] Appellant raises several issues stated in different ways. We find two issues to be dispositive and thus do not find it necessary to discuss the others in detail. She contends, and we conclude: (1) The speed trap laws required the prosecution to establish that the posted speed limit was justified by a valid engineering and traffic survey, and (2) The posted speed limit of 35 mph was not justified by the engineering and traffic survey. D. Overview In this case, there is no issue regarding timing a vehicle over a measured distance, fn. 5 an officer not wearing a distinctive uniform, or an officer using a vehicle not painted a distinctive color. fn. 6 The only issue relates to the use of radar, and therefore the analysis begins with the provisions of subdivision (b) of section 40802. Under subdivision (b) of section 40802, speed trap rules do not apply, and radar can be used: 1. On "local streets and roads" as defined by section 40802, subdivision (b), 2. To enforce an absolute 60 or 65 mph speed limit set pursuant to section 22356, and 3. To enforce the absolute 55 mph speed limit established by section 22349, unless a lower speed limit has been purportedly established pursuant to section 22354 or section 22358. (See fn. 7.) [13 Cal.App.4th Supp. 6] None of these exceptions apply to the present case. Under subdivision (b) of section 40802, there can be no prosecution of any charge involving the speed of a vehicle, fn. 7 where enforcement involves the use of radar, except in compliance with speed trap rules, on a particular section of a highway with a prima facie speed limit decreased pursuant to section 22354, 22358, or 22358.3. fn. 8 This case involves a speed limit decreased by local authority pursuant to section 22358, and therefore the prosecution must comply with the speed trap rules. E. Speed Trap Rules Deputy Berg testified that he visually estimated appellant's speed at 50 to 55 mph in a posted 35 mph zone, and that his radar showed 52 mph. Appellant argues that the prosecution used radar evidence to convict her, the radar evidence was illegally obtained by use of a speed trap, and therefore the court lacked jurisdiction. Appellant cites People v. Halopoff, supra, 60 Cal.App.3d Supp. 1 and People v. Sterritt (1976) 65 Cal.App.3d Supp. 1 [135 Cal.Rptr. 522], among other cases. Respondent contends that in those cases, the only evidence of guilt was a radar reading with no independent speed observations by the officers, whereas in this case the evidence includes an initial visual estimate of appellant's speed which was then confirmed by radar. We are aware that, in unpublished decisions, other panels of this appellate department have distinguished Halopoff and Sterritt on that basis. We conclude that such a distinction is in error. Sections 40802, subdivision (b) and [13 Cal.App.4th Supp. 7] 40803, subdivision (b) and both apply "where enforcement involves the use of radar." (Italics added.) These sections do not say that they apply only where enforcement is exclusively based on the use of radar. Section 40801 prohibits use of a speed trap in securing evidence of the speed of a vehicle for prosecution under the Vehicle Code. fn. 9 Section 40802 defines a "speed trap." fn. 10 It includes a section of a highway with a prima facie speed limit provided by local ordinance pursuant to section 22358, which speed limit is not justified by an engineering and traffic survey conducted within the five years prior to the date of the alleged violation, and where enforcement involves the use of radar. Section 40803, subdivision (a), provides that no evidence as to the speed of a vehicle shall be admitted in any court upon the trial of any person for an alleged violation of this code when the evidence is based upon or obtained from or by the maintenance of a speed trap. fn. 11 [13 Cal.App.4th Supp. 8] Section 40804 provides that in a prosecution for a charge involving speed, any officer shall be incompetent as a witness if the testimony is obtained by the maintenance of a speed trap. fn. 12 Section 40805 provides that a court is without jurisdiction to render a judgment of conviction for speeding if the court admits any evidence secured in violation of sections 40800 through 40808. fn. 13 The only exception for a highway with a prima facie speed limit is for "local streets and roads" as defined in section 40802. Thus, as noted above, section 40802, subdivision (b) provides that a "speed trap" is a section of highway with a prima facie speed limit which is not justified by an engineering and traffic survey conducted within five years and where enforcement involves the use of radar. Subdivision (b) of section 40803 provides that in a speeding prosecution where enforcement involves the use of radar, the prosecution shall establish, as part of its prima facie case, that the evidence is not based upon a speed trap. As in this case, People v. Peterson (1986) 181 Cal.App.3d Supp. 7 [226 Cal.Rptr. 544] and People v. DiFiore (1987) 197 Cal.App.3d Supp. 26 [243 Cal.Rptr. 359] applied the speed trap sections to cases in which officers also [13 Cal.App.4th Supp. 9] testified to visual estimates. (See also People v. Sullivan, supra, 234 Cal.App.3d at pp. 60-62.) Section 40803, subdivision (b) places on the prosecution the duty to establish that the evidence or testimony is not based upon a speed trap, that is, that the speed limit is justified by an engineering and traffic survey. fn. 14 F. The Rules for a Survey fn. 15 In this case, there is a survey which was made within five years prior to the alleged violation. fn. 16 Appellant contends that the survey does not justify the 35 mph speed limit. Can there by a good speed trap when there is a survey within the specified five-year period? Yes. Evidence that there was a survey within five years is prima facie evidence that the evidence or testimony is not based on a speed trap. (§ 40803, subd. (c)). However, that is merely a prima facie case, and the speed limit must be justified by the survey. A speed limit is not justified by a survey unless the survey proves or shows the speed limit to be just and based upon a sufficient lawful reason. What are the rules applicable to a survey which can justify a reduced speed limit for the purpose of radar speed enforcement? 1. A local authority may, based on a survey, set a prima facie speed limit (less than 55 mph) which is most appropriate to facilitate the orderly movement of traffic and is reasonable and safe. fn. 17 This general standard is given more specific meaning by the Department of Transportation. Section [13 Cal.App.4th Supp. 10] 627 provides that a survey must comply with methods determined by the Department of Transportation, and shall consider prevailing speeds, accident records, and conditions not readily apparent to the driver. fn. 18 2. In the absence of other factors, physical conditions readily apparent to a driver do not require reduced speed zoning. fn. 19 3. Methods required by the Department of Transportation are published in a traffic manual. Chapter 8 provides traffic regulations. Sections 8- 03.1 through 8-03.4 deal with speed limits and zones. Excerpts of section 8-03.3 governing establishment of prima facie speed zones are set forth in the appendix. The following except from the traffic manual, section 8-03.3, subdivision B.1.b., provides a frame of reference: "Speed limits should be established preferably at or near the 85 percentile speed, which is defined as that speed at or below which 85 percent of the traffic is moving. ... Speed limits higher than the 85 percentile are not generally considered reasonable and safe and limits below the 85 percentile do not facilitate the orderly movement of traffic. Speed limits established on this basis conform to the consensus of those who drive highways as to what speed is reasonable and safe; and are not dependent on the judgement of one or a few individuals. "The basic speed law states that no person shall drive at a speed greater than is reasonable or prudent. The majority of drivers comply with this law, and disregard regulations which they consider unreasonable. It is only the top fringe of drivers that are inclined to be reckless and unreliable, or who have faulty judgement and must be controlled by enforcement. Speed limits set at or slightly below the 85 percentile speed provide law enforcement [13 Cal.App.4th Supp. 11] officers with a means of controlling the drivers who will not conform to what the majority considers reasonable and prudent. "Only when roadside development results in traffic conflicts and unusual conditions which are not readily apparent to drivers, are speed limits somewhat below the 85 percentile warranted." For the purposes of this case, the rules are well summarized in the traffic manual, section 8-03.3, subdivision B.2.b., which provides in part: "The speed limit normally should be established at the first five mile per hour increment below the 85 percentile speed. However, in matching existing conditions with the traffic safety needs of the community, engineering judgement may indicate the need for a further reduction of five miles per hour. The factors justifying such a further reduction are the same factors mentioned above. Whenever such factors are considered to establish the speed limit, they should be documented on the speed zone survey or the accompanying engineering report. "The Engineering and Traffic Survey should contain sufficient information to document that the conditions of CVC Section 627 have been complied with and that other conditions not readily apparent to a motorist are properly identified. "The establishment of a speed limit of more than 5 miles per hour below the 85 percentile (critical) speed should be done with great care as this may make violators of a disproportionate number of the reasonable majority of drivers." G. Application of the Rules to the Survey in This Case Deputy Berg testified appellant was traveling near a senior citizens' complex where there is often a lot of foot traffic and bicyclists. An officer's description of conditions at the time of the alleged violation would be relevant to whether there was a violation of section 22350, if he were competent to testify. But his testimony is irrelevant to the existence of a speed trap. The existence of a speed trap depends on whether the survey justified the action of the local authority in setting the speed limit. The survey was performed May 19, 1988, well within the five-year requirement. The road is a major artery with a raised median center divider, with two striped lanes in each direction. For traffic in the direction appellant was traveling, the 85th percentile critical speed was 48 mph. Only 15 percent of drivers exceed 48 mph, [13 Cal.App.4th Supp. 12] whereas 85 percent drove at that speed or slower. As a general rule, that would support a prima facie speed limit of 45 mph. The average speed was 43 mph. The speed limit was actually set at 35 mph, a speed exceeded by 95 percent of the drivers. Obviously the collective judgment of the presumed reasonable and prudent majority of drivers does not support the speed limit based on readily apparent conditions. To support such a reduced speed limit, the survey must contain sufficient information to document other conditions not readily apparent to a motorist. The "Accident and Roadside Features Review" section of the survey lists "Unusual Roadside Features" as "1,000' radius curve, limited sight distance to commercial driveways." The segment length is 3,300 feet. The 1987 average daily traffic is stated as 12,300. The number of speed-related accidents (1985, 1986, 1987) is zero. Section 22358.5 precludes justifying reduced speed zoning on physical conditions such as curvature or any other condition readily apparent to a driver. That leaves only the reference to "limited sight distance to commercial driveways" to justify the speed limit. It is questionable whether, with that volume of daily traffic, a condition not apparent to drivers can justify a 10 mph speed reduction unless the accident rate is greater than would be statistically expected from the traffic volume and road type. Here, there were no speed-related accidents within three years. However, the stated condition fails to justify the speed for another reason. The survey does not state the sight distance or the location of the driveways, or explain how the condition affects the safe speed. This is not a mere technical nicety. In this case, two licensed traffic engineers testified there are no driveways that affect safety for traffic traveling in the direction of appellant. The two traffic engineers gave compelling testimony explaining the insufficiencies of the engineering and traffic survey in this case. We have not dwelt on their testimony for several reasons. A trier of fact may, at least under certain circumstances, reject the testimony of expert witnesses. (People v. Green (1984) 163 Cal.App.3d 239 [209 Cal.Rptr. 255].) Most drivers cited for traffic violations post and forfeit bail, feeling they cannot afford the inconvenience or the time off work to contest even a citation they believe to be unfair. Conviction of common, frequent traffic infractions, with the attendant consequences of fines, points toward suspension of driver's licenses, and increased insurance rates, ought not depend on the ability of a [13 Cal.App.4th Supp. 13] driver to obtain the assistance of a licensed traffic engineer. The Legislature has carefully constructed the speed trap laws to be jurisdictional in nature. The prosecution ought not attempt to invoke, and the judiciary ought not attempt to exercise, jurisdiction contrary to the clearly expressed statutory limitation. In the supplemental brief we requested, respondent argues that the survey states the opinion of the city traffic engineer that the accident or roadside features warrant additional speed zone reduction. Respondent concludes: "The established speed limit of 35 miles per hour was based upon a proper compliance with procedure and the law. Although experts in traffic engineering may disagree with a specific speed or conditions, the law requires only that a proper procedure be followed to establish a given speed in a given location." Disagreement of experts will not necessarily invalidate a prima facie speed limit. But if respondent is arguing that an engineer's stated opinion is merely a procedural prerequisite not subject to judicial review, we disagree. A trial judge must first see if there is a timely survey that purports to justify the speed limit. If so, the trial judge must determine if the facts stated in the survey justify the speed limit set. If the judge determines that the speed limit is not justified, the speeding charge must be dismissed. If the judge determines the speed limit is justified, the judge will then decide whether guilt is proved beyond a reasonable doubt, subject to review on both issues if there is a conviction. fn. 20 H. Effect of Proposition 8 Respondent contended (1) speed trap rules were not applicable because there was evidence of a visual estimate of speed, and (2) the speed trap rules, if applicable, were satisfied because there was a survey within five years. Respondent did not address the application of Proposition 8. [3] Having rejected those two arguments by respondent, we must address Proposition 8. The long-standing rule of section 40803, subdivision (a) requiring exclusion of evidence obtained by use of a speed trap was abrogated June 8, 1982, [13 Cal.App.4th Supp. 14] by the adoption of the Proposition 8 "Right to Truth-in-Evidence" provision contained in article I, section 28, subdivision (d), of the California Constitution. (People v. Sullivan, supra, 234 Cal.App.3d 56.) However, as discussed above in part E, the sanction for violation of speed trap prohibitions is not merely exclusion of the offending radar evidence. The officer is incompetent as a witness to a charge of speeding (§ 40804), and the court is deprived of jurisdiction (§ 40805). Some have argued that Proposition 8 also abrogated those provisions. People v. Munoz (1992) 11 Cal.App.4th 1190, 1191 [15 Cal.Rptr.2d 21], footnote 1, indicates that the appellate department, following People v. Sullivan, supra, 234 Cal.App.3d 56, held Proposition 8 eliminated exclusion of speed trap evidence in speeding cases. We question that conclusion. fn. 21 The question has been decisively resolved by the adoption of section 40808 by Statutes 1992, chapter 538, section 2. The measure passed the Senate 35 to 0 and the Assembly 57 to 3, both well in excess of the two- thirds majority required by Proposition 8. The new section, effective January 1, 1993, provides: "Subdivision (d) of Section 28 of Article I of the California Constitution shall not be construed as abrogating the evidentiary provisions of this article." People v. Munoz, supra, 11 Cal.App.4th 1190 held that the new statute should be applied retrospectively to all cases not final on the effective date of the statute. (Chapter 538 also amended section 40803, subdivision (a) to make it, like sections 40803, subdivision (b), 40804, and 40805, applicable only in a prosecution upon a charge involving the speed of a vehicle, preserving the holding of Sullivan in prosecutions of charges not involving speed.) In short, the speed trap rules apply only to charges involving the speed of a vehicle, and are not abrogated by Proposition 8. I. Conclusion The Legislature has spoken clearly on the subject of speed traps. Speed traps-reduced speed zones not justified by the conditions-bring disrespect [13 Cal.App.4th Supp. 15] to law enforcement and the courts. We have discussed the requirements and consequences at length because it must be clear to traffic engineers, local authorities, and law enforcement officers that if a prima facie speed limit is set without being justified in fact by the engineering and traffic survey, the speed limit cannot be enforced by any means involving the use of radar. Local authorities must set prima facie speed limits carefully, as justified by appropriate factors, to avoid making use of radar unavailable for speed enforcement. When enforcing traffic laws by plain sight patrolling, officers should exercise their law enforcement discretion based on the same reasonable and prudent judgments as most other drivers, not influenced by prima facie speed zones which are not justified. [2b] The survey in this case did not justify the prima facie speed limit. Enforcement involved the use of radar. Thus, a speed trap existed. The officer was therefore not competent as a witness and the court was without jurisdiction to render the judgment of conviction. The judgment is reversed, and the case is remanded to the municipal court with directions to dismiss. McNally, J., and Steele, J., concurred. [13 Cal.App.4th Supp. 16] TRAFFIC MANUAL CHAPTER 8 - REGULATIONS Speed Limits and Zones 8-03 * * * 8-03.3 Establishment of Prima Facie ... Speed Zones A. Legal Authority * * * 7. Speed Trap - Section 40802(b) provides that prima facie speed limits established under Sections 22352(b)(1), 22354, 22357, 22558 and 22358.3 may not be enforced by radar unless the speed limit has been justified by an engineering and traffic survey within the last five years. An "Engineering and Traffic Survey" is required where enforcement involves the use of radar or other electronic speed measuring devices, under CVC 40802(b). Local streets and roads, as defined in the second paragraph of CVC 40802(b), primarily serving abutting residential property, are exempt from this requirement .... B. Engineering and Traffic Surveys Section 627 of the Vehicle code defines the term "Engineering and Traffic Survey" and lists requirements therefor. Following are two methods of conducting engineering and traffic surveys to be used to establish or justify prima facie speed limits. These methods are presented as required by the Vehicle Code. 1. State Highways - The engineering and traffic survey for State highways is made under the direction of the District Traffic Engineer. The data shall include: a. One copy of the Standard Speed Zone Survey Sheet showing: . A north arrow. . Engineer's station or post mileage. . Limits of the proposed zones. . Appropriate notations showing type of roadside development, such as "scattered business", "solid residential", etc. Schools adjacent to the highway should be shown, but other buildings need not be plotted unless they are a factor in the speed recommendation or the point of termination of a speed zone. . Accident rates for the zones involved. . Average daily traffic volume. . Location of traffic signals, signs and markings. . If the highway is divided, the limits of zones for each direction of travel. . Plotted 85 percentile and pace speeds at location taken showing speed profile. b. A report to the District Director shall: . State the reason for the initiation of speed zone survey. . Give recommendations and reasons therefor. . List the enforcement jurisdictions involved and the attitude of these officials. . Give the stationing or mileage at the beginning and at the end of each proposed zone and any intermediate equations. Ties must be given to readily identifiable physical features. In determining the speed limit which is most appropriate to facilitate the orderly movement of traffic and is reasonable and safe, important factors are prevailing speeds, unexpected conditions, and accident records. [13 Cal.App.4th Supp. 17] Speed limits should be established preferably at or near the 85 percentile speed, which is defined as that speed at or below which 85 percent of the traffic is moving. The 85 percentile is often referred to as critical speed. Pace speed is defined as the 10-mile increment of speed containing the largest number of vehicles. The lower limit of the pace is plotted on the Speed Zone Survey Sheets as an aid in determining the proper zone limits. Speed limits higher than the 85 percentile are not generally considered reasonable and safe and limits below the 85 percentile do not facilitate the orderly movement of traffic. Speed limits established on this basis conform to the consensus of those who drive highways as to what speed is reasonable and safe; and are not dependent on the judgment of one or a few individuals. The basic speed law states that no person shall drive at a speed greater than is reasonable or prudent. The majority of drivers comply with this law, and disregard regulations which they consider unreasonable. It is only the top fringe of drivers that are inclined to be reckless and unreliable, or who have faulty judgment and must be controlled by enforcement. Speed limits set at or slightly below the 85 percentile speed provide law enforcement officers with a means of controlling the drivers who will not conform to what the majority considers reasonable and prudent. Only when roadside development results in traffic conflicts and unusual conditions which are not readily apparent to drivers, are speed limits somewhat below the 85 percentile warranted. Concurrence and support of enforcement officials are necessary for the successful operation of a restricted speed zone. Section 22358.5 of the Vehicle Code states that it is the intent of the Legislature that physical conditions such as width, curvature, grade and surface conditions, or any other condition readily apparent to the driver, in the absence of other factors, would not require special downward speed zoning. Speed zones of less than half a mile and short transition zones should be avoided. 2. City and County Through Highways, Arterials, Collector Roads and Local Streets. a. Introduction - This is a short method of speed zoning based on the premise that a reasonable speed limit is one that conforms to the actual behavior of the majority of motorists, and that by measuring motorists' speeds, one will be able to select a speed limit that is both reasonable and effective. Other factors that need to be considered are the most recent two year accident record, roadway design speed, safe stopping sight distance, superelevation, shoulder conditions, profile conditions, intersection spacing and offsets, commercial driveway characteristics, pedestrian traffic in the roadway without sidewalks, etc. In most situations, the short form will be adequate, but the procedure used on State highways may be used at the option of the agency. b. Determination of Existing Speed Limits - These speeds will either be verified, increased or decreased depending on the results of the investigation. Specific types of vehicles may be tallied by use of letter symbols in appropriate squares. The speed limit normally should be established at the first five mile per hour increment below the 85 percentile speed. However, in matching existing conditions with the traffic safety needs of the community, engineering judgment may indicate the need for a further reduction of five miles per hour. The facts justifying such a further reduction are the same factors mentioned above. Whenever such factors are considered to establish the speed limit, they should be documented on the speed zone survey or the accompanying engineering report. The Engineering and Traffic Survey should contain sufficient information to document that the conditions of CVC Section 627 have been complied with and that other conditions not readily apparent to a motorist are properly identified. The establishment of a speed limit of more than 5 miles per hour below the 85 percentile (critical) speed should be done with great care as this may make violators of a disproportionate number of the reasonable majority of drivers. [13 Cal.App.4th Supp. 18] c. Speed Zone Survey - . The intent of the speed measurements [is] to determine the actual speed of the unimpeded traffic. The speed of traffic should not be altered by concentrated law enforcement, or other means, just prior to, or while taking the speed measurements. . Only one person is required for the field work. Speeds can be read directly from a radar meter. . Devices, other than radar, capable of accurately distinguishing and measuring the unimpeded speed of free flowing vehicles unaffected by platoon movement may be used. Special application of devices other than radar are particularly appropriate on low volume facilities. . A location should be selected where prevailing speeds are representative of the entire speed zone section. If speeds vary on a given route, more than one speed zone section may be required, with separate measurements for each section. Locations for measurements should be chosen so as to minimize the effects of traffic signals or stop signs. . Speed measurements should be taken during off-peak hours on weekdays. If there is difficulty in obtaining the desired quality, speed measurements may be taken during any period with free flowing traffic. The weather should be fair with no unusual conditions prevailing. It is important that the surveyor and his equipment be so inconspicuous as not to affect the traffic speeds. For this reason an unmarked car is recommended, with radar speed meter located as inconspicuously as possible. It should be placed so as to be able to survey traffic in both directions, and should not make an angle greater than 15 degrees with the roadway centerline. . In order for the sample to be representative of the actual traffic flow, it is desirable to have a minimum sample of 100 vehicles in each survey. In no case should the sample for any survey contain less than 50 vehicles. . Short speed zones of less than half a mile should be avoided, except in transition areas. . Speed zone changes should be coordinated with changes in roadway conditions or roadside development. . Speed zoning should be in 10 mile per hour increments except in urban areas where 5 mile per hour increments are preferable. . Speed zoning should be coordinated with adjacent jurisdictions. [Figures omitted.] ­FN 1. "No person shall drive a vehicle upon a highway at a speed greater than is reasonable or prudent having due regard for weather, visibility, the traffic on, and the surface and width of, the highway, and in no event at a speed which endangers the safety of persons or property." (§ 22350.) ­FN 2. Unless otherwise indicated, all section references are to the Vehicle Code. ­FN 3. The abbreviation "mph" will often be used herein in place of "miles per hour." ­FN 4. Since 95 percent of drivers exceeded the prima facie speed limit, it is evident that the excellent safety record on that segment of the roadway was not the result of any reduced speed limit, but was the result of the road conditions and safe driving uninfluenced by the low speed limit. ­FN 5. Calculating speed of a vehicle by timing it over a measured distance is defined as a "speed trap" by section 40802, subdivision (a). Enforcement is provided by sections 40801, 40803, subdivision (a), 40804, subdivision (a), and 40805. ­FN 6. The uniform and vehicle requirements are specified in section 40800, with enforcement provided by section 40804, subdivision (b). ­FN 7. If radar is used in conjunction with a prima facie speed limit, the prima facie speed limit must be justified as required by the statute even if the driver is charged with violation of the 55 mph maximum speed limit under section 22349 and is not charged with violation of the prima facie speed limit. (People v. Flaxman (1977) 74 Cal.App.3d Supp. 16, 18- 19 [141 Cal.Rptr. 799].) ­FN 8. It would be helpful to have legislation clarifying whether section 40802, subdivision (b)'s reference to a "section of a highway with a prima facie speed limit provided by this code" includes speed limits: 1. Set by section 22352, subdivision (a) at 15 mph at railway grade crossings with obstructed views, intersections with obstructed views, and alleys; 2. Set by section 22352, subdivision (b) at 25 mph in a business or residence district, when passing a school while children are going or coming during school hours, and when passing a posted senior center; 3. Increased pursuant to section 22357; 4. Made variable for a freeway pursuant to section 22355; 5. Set pursuant to section 22357.1 at 25 mph adjacent to a children's playground in a public park during particular hours; or 6. Reduced to 20 or 15 mph near a school or senior center, pursuant to section 22358.4. ­FN 9. "No peace officer or other person shall use a speed trap in arresting, or participating or assisting in the arrest of, any person for any alleged violation of this code nor shall any speed trap be used in securing evidence as to the speed of any vehicle for the purpose of an arrest or prosecution under this code." (§ 40801.) ­FN 10. "A speed trap is either of the following: "(a) A particular section of a highway measured as to distance and with boundaries marked, designated, or otherwise determined in order that the speed of a vehicle may be calculated by securing the time it takes the vehicle to travel the known distance. "(b) A particular section of a highway with a prima facie speed limit provided by this code or by local ordinance pursuant to paragraph (1) of subdivision (b) of Section 22352, or established pursuant to Section 22354, 22357, 22358, or 22358.3, which speed limit is not justified by an engineering and traffic survey conducted within five years prior to the date of the alleged violation, and where enforcement involves the use of radar or other electronic devices which measure the speed of moving objects. This subdivision does not apply to local streets and roads. "For purposes of this section, local streets and roads shall be defined by the latest functional usage and federal-aid system maps as submitted to the Federal Highway Administration. When these maps have not been submitted, the following definition shall be used: A local street or road primarily provides access to abutting residential property and shall meet the following three conditions: "(1) Roadway width of not more than 40 feet. "(2) Not more than one-half mile of uninterrupted length. Interruptions shall include official traffic control devices as defined in Section 445. "(3) Not more than one traffic lane in each direction." (§ 40802.) ­FN 11. "(a) No evidence as to the speed of a vehicle upon a highway shall be admitted in any court upon the trial of any person for an alleged violation of this code when the evidence is based upon or obtained from or by the maintenance or use of a speedtrap. "(b) In any prosecution under this code of a charge involving the speed of a vehicle, where enforcement involves the use of radar or other electronic devices which measure the speed of moving objects, the prosecution shall establish, as part of its prima facie case, that the evidence or testimony presented is not based upon a speedtrap as defined in subdivision (b) of Section 40802. "(c) When a traffic and engineering survey is required pursuant to subdivision (b) of Section 40802, evidence that a traffic and engineering survey has been conducted within five years of the date of the alleged violation or evidence that the offense was committed on a local street or road as defined in subdivision (b) of Section 40802 shall constitute a prima facie case that the evidence or testimony is not based upon a speedtrap as defined in subdivision (b) of Section 40802." (§ 40803.) Note: Section 40803 has since been amended by Statutes 1992, chapter 538, section 1, effective January 1, 1993. ­FN 12. "(a) In any prosecution under this code upon a charge involving the speed of a vehicle, any officer or other person shall be incompetent as a witness if the testimony is based upon or obtained from or by the maintenance or use of a speed trap. "(b) Every officer arresting, or participating or assisting in the arrest of, a person so charged while on duty for the exclusive or main purpose of enforcing the provision of Divisions 10 and 11 is incompetent as a witness if at the time of such arrest he was not wearing a distinctive uniform, or was using a motor vehicle not painted the distinctive color specifically by the commissioner. "This section does not apply to an officer assigned exclusively to the duty of investigating and securing evidence in reference to any theft of a vehicle or failure of a person to stop in the event of an accident or violation of Section 23109 or in reference to any felony charge or to any officer engaged in serving any warrant when the officer is not engaged in patrolling the highways for the purpose of enforcing the traffic laws." (§ 40804.) ­FN 13. "Every court shall be without jurisdiction to render a judgment of conviction against any person for a violation of this code involving the speed of a vehicle if the court admits any evidence or testimony secured in violation of, or which is inadmissible under this article." (§ 40805.) ­FN 14. People v. Halopoff, supra, 60 Cal.App.3d Supp. 1, and People v. Sterritt, supra, 65 Cal.App.3d Supp. 1, 6, footnote 4, held that to avoid the consequences of a speed trap finding, the prosecution must produce the engineering and traffic survey. The requirements of Halopoff and Sterritt were codified in 1981 as subdivision (b) of section 40803. That provision was interpreted and applied in People v. Peterson, supra, 181 Cal.App.3d Supp. 7. ­FN 15. Subdivision (c) of section 40803 refers to a "traffic and engineering survey." Subdivision (b) of section 40802 and other sections refer to an "engineering and traffic survey." They obviously refer to the same thing, which we shall generally refer to as a survey. ­FN 16. Appellant raises the issue whether the existence of the survey was sufficiently before the court. The officer testified to a survey, appellant produced two expert witnesses who testified to details of the same survey, and respondent has provided a certified copy of a survey which is obviously the same one relied on by the prosecution in trial and attacked by appellant at trial and on appeal. We elect to deal with the merits of the survey rather than remand for retrial merely to have the survey clearly identified in the record. ­FN 17. "Whenever a local authority determines upon the basis of an engineering and traffic survey that the limit of 55 miles per hour is more than is reasonable or safe upon any portion of any street other than a state highway where the limit of 55 miles per hour is applicable, the local authority may by ordinance determine and declare a prima facie speed limit of 50, 45, 40, 35, 30, or 25 miles per hour, whichever is found most appropriate to facilitate the orderly movement of traffic and is reasonable and safe, which declared prima facie limit shall be effective when appropriate signs giving notice thereof are erected upon the street." (§ 22358.) ­FN 18. "(a) 'Engineering and traffic survey', as used in this code, means a survey of highway and traffic conditions in accordance with methods determined by the Department of Transportation for use by the state and local authorities. "(b) An engineering and traffic survey shall include, among other requirements deemed necessary by the department, consideration of all of the following: "(1) Prevailing speeds as determined by traffic engineering measurements. "(2) Accident records. "(3) Highway, traffic, and roadside conditions not readily apparent to the driver." (§ 627.) ­FN 19. "It is the intent of the Legislature that physical conditions such as width, curvature, grade and surface conditions, or any other condition readily apparent to a driver, in the absence of other factors, would not require special downward speed zoning, as the basic rule of Section 22350 is sufficient regulation as to such conditions." (§ 22358.5.) ­FN 20. Review at trial or on appeal will tend to fall into patterns. Many speed limits will apparently be justified because, in accordance with the general rule, they are set at the 85th percentile speed or within 5 mph under that speed. Some speed limits may be justified because they are set five mph below the general rule, based on higher than expected accident rates or listed hidden hazards. Some speed limits may appear to be unjustified or questionable because: 1. The speed limit is set 10 or more mph under the 85th percentile speed; 2. The speed limit makes violators of a large percentage of drivers; 3. "Conditions" listed are not hidden hazards, that is, they are readily apparent to a driver; 4. There is no explanation how the conditions listed require the speed limit set; or 5. The accident rate is not higher than would be expected statistically. ­FN 21. Sullivan involved only section 40803, subdivision (a) which excluded speed trap evidence in all Vehicle Code prosecutions, including driving under the influence of alcohol as alleged in that case, Sullivan, supra, 234 Cal.App.3d at page 63, held that "... section 40803, subdivision (a), can only be characterized as a rule of evidence." The court continued: "The Legislature could have selected other remedies or penalties for violation of section 40801, but it clearly opted for the exclusion of evidence." (Ibid.) That remedy was abrogated by Proposition 8. But the court stated: "Sections 40803, subdivision (b), 40804, and 40805, which apply only when a defendant is charged with an offense involving the speed of a vehicle, do not apply in this case." (Id., at p. 60.) Those sections do provide other remedies or penalties for violation of section 40801 besides exclusion of evidence. We doubt that Proposition 8 was intended to abrogate statutes making a person incompetent as a witness and limiting the jurisdiction of a court.
Editor's note: The CalTrans Traffic manual has been replaced by the Manual on Uniform Traffic Control Devices (MUTCD) 2003 as amended by the MUTCD 2003 California Supplement (see Chapter 2B: Regulatory Signs; pages 51 - 54). This is important as the language was changed from the nearest 5 mph increment below the 85th percentile to the nearest 5mph increment to the 85th percentile.

16 comments:

  1. As of today, September 30th, has TN done the engineering and traffic investigation?

    After years of riding the area as a visitor, I recently moved to TN and within 30 minutes from Deals Gap. For the first time since 2007, I was ticketed for speeding. Do I have a leg to stand on or is the speed limit still defaulted to 65?

    Responses can be sent to me directly at islndgal@cxp.com

    ReplyDelete
  2. ECU Remaps have been vehicle tuners all our lives, ask us a complex mechanical question and ECU Remaps will be able to answer it. Ask us to strip an engine bare and rebuild it in half a day after blowing it up on a rally stage and ECU Remaps will do it. ECU Remaps have over 50+ years combined motorsport experience.All of ECU Remaps remapping are based on tunes ECU Remaps have produced on a 3000bhp, 4x4, state of the art Mustang, motorsport dyno ECU Remaps have shared access to.

    ReplyDelete
  3. Have they done the engineering and traffic investigation as of yet? I was there on Aug 6th. 2017 and received a ticket which states I was doing 53MPH in a "30MPH" zone. Should I fight this ticket or not I'm also from Canada so not entirely sure how it all works.

    ReplyDelete
  4. thank you for sharing the information with us. kindly share speed display.

    ReplyDelete
  5. Such a nice blog with useful information. I would be thankful if you share more information about Speeding Ticket Lawyer .
    Speeding Ticket Lawyer

    ReplyDelete
  6. thank you for sharing information with us. I really like information regardingradar speed sign, speed display, speed display signs keep sharing it.

    ReplyDelete
  7. Idk u must ask tdot legal dept at straberry plains. Probably not, its NEVER done anywhere, if so speed limits must be massively raised like in knoxville today, 55 raised to 70, millions of bogus speeding tickets invalid for 40 years

    https://www.wbir.com/mobile/article/traffic/speed-limit-on-i-40-i-75-i-640-in-knoxville-now-65-mph-in-some-places/51-9ab6df16-27cd-4015-995e-264528649751

    ReplyDelete
  8. This comment has been removed by the author.

    ReplyDelete
  9. This comment has been removed by the author.

    ReplyDelete


  10. https://www.wbir.com/mobile/article/traffic/speed-limit-on-i-40-i-75-i-640-in-knoxville-now-65-mph-in-some-places/51-9ab6df16-27cd-4015-995e-264528649751

    Speed limits increase on I-40, I-75, I-640 in Knoxville overnight

    There are now 65 mph speed limit signs on I-40 and I-640 in the Knoxville area.

    Lauren Hoar, WBIR Staff

    October 21, 2019

    KNOXVILLE, Tenn. — You may have noticed some new signs along the interstate on your way to work Monday morning. 

    You aren't crazy. The speed literally went up overnight from 55 mph to 65 mph on most of I-40 in West Knoxville.  The rest of I-40 and 640 through Knox County will have the posted speed limit changed Monday night to 60, 65, and 70 mph.

    Speed limit changes on I-40 and I-640 in Knox and Sevier Counties.

    The change in speed on the ground is made possible by improvements to the quality of the air.

    The more you put your foot on the gas, the more emissions your car pumps into the air. That's why Knoxville's speed was kept at 55 mph for many years.

    TVA's installation of scrubbers to reduce air pollution at coal-powered plants have drastically increased air quality in East Tennessee.

    TDOT Spokesperson Mark Nagi issued the following statement Monday: 

    "TDOT has full authority to establish regulatory speeds on controlled access facility. Speed limits should be safe and reasonable, meet drivers' expectations, consider operational speeds, critical speed (15 mph over limit), reflect design speed, and consider crash data. Back in 2004 TDOT established a policy of reducing speed limits to improve air quality. At the request of local governments in Knoxville area counties, we agreed to reduce speed limits and interstate speed limits in the Knoxville area were changed from 70 to 65/55 split due to the air qualify nonattainment status for ozone."

    Mark Nagi@MarkNagiTDOT

    You’ll notice some speed limit changes occurring on I-40 and on I-640 in the Knoxville area. Signage started to be installed last night with more of this work taking place this evening on I-40 between the I-640 splits, as well as I-640 itself.

    Nagi said since 2004, the Knoxville area has regained full compliance with all current National Ambient Air Quality Standards as regulated by the Clean Air Act. As a result, Nagi said TDOT initiated a review of posted speed limits on the Knoxville area interstates with support from the Knoxville Regional Transportation Planning Organization. 

    "We found that 87%-98% of motorists were already going over the 55 mph speed limit.  According to the Manual on Uniform Traffic Control Devices (MUTCD), the posted speed limit should be generally within 5 mph of the 85th percentile speed, which we found to be between 67-75 mph in the Knoxville area, regardless of the posted speed limit. In some spots over 25% of all traffic was traveling above that critical speed (15 mph over limit)."

    According to crash data, Nagi said with the exception of a part of I-40 in downtown Knoxville, crash rates are below the statewide average in the study area.

    https://www.wbir.com/mobile/article/traffic/speed-limit-on-i-40-i-75-i-640-in-knoxville-now-65-mph-in-some-places/51-9ab6df16-27cd-4015-995e-264528649751

    ReplyDelete
  11. The more u speed the safer u get, sats tdot:

    According to crash data, Nagi said with the exception of a part of I-40 in downtown Knoxville, crash rates are below the statewide average in the study area.

    https://www.wbir.com/mobile/article/traffic/speed-limit-on-i-40-i-75-i-640-in-knoxville-now-65-mph-in-some-places/51-9ab6df16-27cd-4015-995e-264528649751

    ReplyDelete
  12. You aren't crazy. Weve just been gaslighting you dumb lazy ignorant suckers for 50 years, says WBIR TV, TDOT and THP. Just like gasoline prices used to be 10 cents a gallon, and still are around the world, and could be in USA today.

    https://www.wbir.com/mobile/article/traffic/speed-limit-on-i-40-i-75-i-640-in-knoxville-now-65-mph-in-some-places/51-9ab6df16-27cd-4015-995e-264528649751

    ReplyDelete
  13. Good post related to signs. Speed limit signs are very important near school and hospital. And speed limit is also used in high accident zones. We are dealers of safety signs and was very good to read this post. Keep writing more on other signs also.

    ReplyDelete
  14. I like the helpful information you provide in your articles. I will bookmark your blog and check again here regularly. I am quite certain I will learn a lot of new stuff right here! Best of luck for the next!

    ReplyDelete